Question of the Day (From Core Concepts Anesthesia Review + Open Anesthesia) + Apex Review Questions

Réussis tes devoirs et examens dès maintenant avec Quizwiz!

A nasopharyngeal airway should be avoided in all of the following circumstances except: - Coagulopathy - Previous transphenoidal hypophysectomy - Basilar skull fracture - Light Anesthesia

- Light Anesthesia

Which characteristics predict difficulty with laryngoscopy? (Select 3) - Long upper incisors - Prognathism - Arched Palate - Mandibular protrusion test class 3 - Long Neck - Cormack-Lehane class 4

- Long upper incisors - Arched Palate - Mandibular protrusion test class 3

Which of the following is most likely to injure the left RLN while sparing the right RLN? - external pressure from a LMA - external pressure from a ETT - Mitral stenosis - Parathyroidectomy

- Mitral stenosis

Identify the best example of acute intrinsic restrictive lung disease. - Negative pressure pulmonary edema - Ankylosing spondylitis - Amiodarone induced pulmonary fibrosis - Pregnancy

- Negative pressure pulmonary edema

Select the statement that best describes hypoxic pulmonary vasoconstriction. - Low PaO2 causes pulmonary vasoconstriction - 1.5 MAC desflurane stimulates pulmonary vasoconstriction - It achieves maximum effect after 2 hours - Nitroprusside increases venous admixture

- Nitroprusside increases venous admixture

To ensure the best chance for successful tracheal intubation, which axes should be aligned? (Select 3) - Oral - Pharyngeal - Laryngeal - Mandibular - Sternal - Tracheal

- Oral - Pharyngeal - Laryngeal

Which of the following is indicated immediately after non particulate pulmonary aspiration? - PEEP - Bronchoscopy - Hydrocortisone - Clindamyacin

- PEEP

What is the consequence of adding a heat and moisture exchanger between the tracheal tube and y-peice. Assume that minute ventilation is held constant (select 2) - PaCO2 decreases - PaCO2 increases - PaO2 decreases - PaO2 remains unchanged

- PaCO2 increases - PaO2 decreases

All of the following variables are required to calculate the partial pressure of alveolar oxygen except: - barometric pressure - PaCO2 - PaO2 - Respiratory quotient

- PaO2

A patient breathing room air has an A-a gradient of 100. Which conditions best explain this finding? (Select 2) - Pulmonary Edema - Pneumonia - Opioid overdose - High altitude

- Pulmonary Edema - Pneumonia

What muscle(s) provide the most significant contribution to forced exhalation? - Diaphragm - Internal Intercostals - Rectus Abdominus - Sternocleidomastoids

- Rectus Abdominus

A patient requires a mediastinoscopy with tracheal lymph node biopsy. Where is the most appropriate place to insert an arterial line to monitor for vascular compression from the mediastinoscope? - Right arm - Right Leg - Left arm - Left leg

- Right arm

Choose the best mechanical ventilation strategy for a patient with severe pulmonary edema. - Tv = 10 ml/kg and RR = 16 - Tv = 6 ml/kg and RR = 8 - Tv = 6 ml/kg and RR = 16 - Tv = 10 ml/kg and RR = 8

- Tv = 6 ml/kg and RR = 16

What is the most common etiology of hypoxemia in the PACU? - Diffusion Limitation - Hypoxic Mixture - Left to Right Shunt - V/Q Mismatch

- V/Q Mismatch

Match each long volume to it's respective components: 1. Inspiratory Capacity 2. Total Lung Capacity 3. Functional Residual Volume 4. Vital Capacity A. ERV + RV B. IRV + Vt + ERV C. IRV + Vt D. IRV + Vt + ERV + RV

1. --> C 2. --> D 3. --> A 4. --> B

A patient is scheduled for a thoracotomy and right upper lobe resection. Which of the following preoperative findings is the best predictor of an increased risk of postoperative pulmonary complications. - DLCO = 47% predicted - VO2 Max = 8 mL 02/kg/min - PaCO2 = 43mmHg - FEV1 = 62% predicted

- VO2 Max = 8 mL 02/kg/min

Connect the correct parings: 1. Oral ETT in women 2. Oral ETT in men 3. Nasal ETT in men 4. Nasal ETT in women A. 25 cm B. 27 cm C. 23 cm D. 21 cm

1. --> D 2. --> C 3. --> B 4. --> A

The lumbar plexus is derived from which of the following? - Dorsal rami of L1-4 - Ventral rami of L1-4 and variable contributions from T12 and L5 - Dorsal rami of L1-4 and variable contributions from S1-2 and L5 - Ventral rami of L1-4 and variable contributions of S1-2 and L5

- Ventral rami of L1-4 and variable contributions from T12 and L5 The lumbar plexus is derived from the anterior (ventral) primary rami of lumbar nerves L1-L4 with variable contributions from the 12th thoracic and 5th lumbar nerves.

All of the following are associated with a reduction in carbon monoxide diffusion capacity in the lung except: - sarcoidosis - emphysema - asthma - pulmonary edema

- asthma

Match each drug with its mechanism of action: 1. Montelukast 2. Theophylline 3. Cromolyn 4. Ipratropium A. Leukotriene antagonist B. Phosphodiesterase inhibitor C. Mast cell stabilizer D. Anticholinergic

1. --> A 2. --> B 3. --> C 4. --> D

Compared to using the Trachlight in the adult population, all of the following are true for its use in children except: - There is a higher incidence of false positive results - It should be bent to at a less acute angle - The bend should be closer to the tip - Transillumination occurs sooner

- It should be bent to at a less acute angle

Which I:E ratio most appropriate for a patient with COPD? - 1:1 - 1:2 - 1:3 - 2:1

- 1:3

Residual volume contributes to what % of TLC? - 10% - 15% - 20% - 25%

- 20%

A patient is scheduled for a left pneumonectomy. Crystalloid administration should be less than: - 3 Liters in 12 hours - 3 Liters in 24 hours - 5 Liters in 12 hour - 5 Liters in 24 hours

- 3 Liters in 24 hours

Which phenomenon is responsible for tachypnea that accompanies PE? - Paradoxical reflex of head - J receptor stimulation - Hering-Breuer inflation reflex - Dive reflex

- J receptor stimulation

In preparation for an awake intubation, you anesthetize the upper airway with aerosolized lidocaine. Shortly after you begin the procedure, the patient is unable to tolerate the scope just beyond the epiglottis but before the vocal cords. Which regional technique will increase the patient's ability to tolerate the rest of the procedure? - 2ml at the tonsillar pillars bilaterally - 3ml at the inferior aspect of the greater cornu of the hyoid bone bilaterally - 4ml at the thyroepiglottic membrane - 5ml at the circothyroid membrane

- 3ml at the inferior aspect of the greater cornu of the hyoid bone bilaterally

Which of the following is MOST true regarding acetaminophen poisoning? - 5% of acetaminophen is excreted in the urine - Acetaminophen is responsible for at least 90% of acute hepatic failure in the US - Acetaminophen is a central COX-1 inhibitor - The majority of acetaminophen is oxidized by NAPQI (which is detoxified by glutathione)

- 5% of acetaminophen is excreted in the urine

Which of the following is an absolute contraindication for circothyroidotomy? - Unstable cervical spine - Neck burn injury - Facial Trauma - Age = 6 years

- Age = 6 years

The first nurse anesthetist to be appointed to a university medical school faculty was: Alice Maude Hunt Agatha Hodgins Helen Lamb Alice Magaw

- Alice Maude Hunt In 1922, Alice Maude Hunt became an instructor in anesthesia at the Yale University School of Medicine and was later promoted to assistant professor. Hunt also pioneered the use of nitrous oxide and oxygen as an anesthetic modality. - Nagelhout Pg. 16

A patient is undergoing thoracotomy in the lateral decubitus position with one lung ventilation. The SpO2 has decreased to 86% and it has not improved with CPAP to the non dependent lung. Which of the following maneuvers should be attempted next? - Ask the surgeon to clamp the pulmonary artery of the non dependent lung. - Increase the tidal volume - Resume two lung ventilation - Apply PEEP to the dependent lung

- Apply PEEP to the dependent lung

An adequacy of the seal created by an LMA classic is most dependent on (select 2): - Cuff pressure - Appropriate sizing - Cuff volume - Correct placement

- Appropriate sizing - Correct placement

Where is the adult larynx located? - C2-C4 - C3-C6 - C4-C7 - C5-T1

- C3-C6

All of the following are contraindications to retrograde intubation except: - Cervical spine injury - Tracheal stenosis - Pretracheal abscess - Morbid obesity

- Cervical spine injury

What is the most reliable sign of endotracheal intubation? - ETT visualization between vocal cords - Chest rise and fall - Mist inside ETT - Bilateral breath sounds

- ETT visualization between vocal cords

All of the following are absolute indications for one lung ventilation except: - Esophageal resection - Isolation of one lung to prevent contamination - Unilateral bronchopulmonary levage - control of distribution of ventilation

- Esophageal resection

Which diagnostic indicator is most indicative a severe asthma attack? - FEV1 = 30% - PaO2 = 65 mmHg - MMEF = 40% - PaCO2 = 45 mmHg

- FEV1 = 30%

In the patient with an unstable cervical spine, which of the following techniques is associated with the least amount of cervical motion? (select 2) - Bullard laryngoscopy - Fiberoptic laryngoscopy - Direct vision laryngoscopy - Blind nasal intubation

- Fiberoptic laryngoscopy - Blind nasal intubation

Which of the following is the gold standard for managing the difficult airway? - Flexible FOB with patient awake and spontaneous ventilation - Video laryngoscope with patient asleep and controlled ventilation - Video laryngoscope with patient asleep and spontaneous ventilation - Flexible FOB with patient asleep and spontaneous ventilation

- Flexible FOB with patient awake and spontaneous ventilation

Which of the following neck positions is associated with the best chance of endobronchial intubation? - Neutral - Flexion - Extension - Neck position does not influence the position

- Flexion

Intrapleural pressure becomes positive during: - Maximum Inspiration - End Expiration - End inspiration - Forced Expiration

- Forced Expiration

Risk factors for intraoperative laryngospam include (SELECT 3) - old age - hypercapnia - GERD - Deep anesthesia - Exposure to 2nd hand smoke - Recent URI

- GERD - Exposure to 2nd hand smoke - Recent URI

Which nerve innervates the region where the MAC blade should be placed during larngyoscopy? - Superior Laryngeal - Recurrent Laryngeal - Glossopharngeal - Trigeminal

- Glossopharngeal

Predictors of difficult mask ventilation include: (Select 3) - History of snoring - Age 50 years - Edentulousness - BMI 25 kg/m2 - Presence of beard - Mallampati 2

- History of snoring - Edentulousness - Presence of beard

A patient in the PACU exhibits signs of recurarization despite reversal with neostigmine and glycopyrrolate. Which of the following conditions will further impair reversal? - Hypercalcemia - Hyperkalemia - Hypoventilation - Hypoalbuminemia

- Hypoventilation

A patient's PaCO2 has increased while her EtCO2 has decreased. All of the following are likely to contribute to this phenomenon except: - hypotension - COPD - Amniotic fluid embolism - Increasing tube length of the circle system

- Increasing tube length of the circle system

All of the following venous systems contribute to anatomic shunt except the: - Pleural veins - Internal thoracic veins - Bronchiolar veins' - Thebesein veins

- Internal thoracic veins (internal mammary veins)

The site of injury in the patient with flail chest moves (select 2): - Inward during inspiration - Inward during expiration - Outward during inspiration - Outward during expiration

- Inward during inspiration - Outward during expiration

Compared to spontaneous ventilation, what happens to the Vd/Vt ratio when the patient is placed on a mechanical ventilator? - increases - decreases - remains the same - There is not enough info to answer question

- It increases

Which airway management technique activates the sympathetic nervous system the least? - Fiberoptic intubation - Combitube - Laryngeal mask airway - Direct vision laryngoscopy

- Laryngeal mask airway

Sinus arrhythmia: - is mediated through sympathetic innervation of the AV node - causes an increase in heart rate with inspiration - is indicative of SA node ischemia - is the primary cause of premature atrial contractions

- causes an increase in heart rate with inspiration - Sinus arrhythmia is a cyclic variation in heart rate that corresponds to ventilation, increasing with inspiration and decreasing with expiration. Sinus arrhythmia is a normal cardiac rhythm and is due to cyclic changes in vagal tone. - Barash pg 1717

All of the following increase the risk complications when an LMA is used except: - an undersized LMA - Nitrous oxide - lateral decubitus position - cuff pressure = 50 cm H2O

- cuff pressure = 50 cm H2O

EMLA cream toxicity: (select two) - increases p50 - decreases p50 - Shift the oxyhemoglobin disassociation curve to the left - Shift the oxyhemoglobin disassociation curve to the right

- decreases p50 - Shift the oxyhemoglobin disassociation curve to the left

Anesthetic considerations for a patient with for cor pulmonale include avoidance of all of the following except: - dexmedetomidine - hypothermia - nitrous oxide - hypoventilation

- dexmedetomidine

All of the following are expected to increase in the patient experiencing a massive pulmonary embolism during surgery except: - deadspace - end tidal carbon dioxide - right ventricular stroke work - heart rate

- end tidal carbon dioxide

Venous admixture increases when: - inspiratory reserve volume increases - inspiratory reserve volume decreases - expiratory reserve volume increases - expiratory reserve volume decreases

- expiratory reserve volume decreases

The circothyroid muscle is innervated by the: - internal branch of the SLN - glossopharyngeal nerve - external branch of SLN - RLN

- external branch of SLN

When compared to emphysema, which of the following most accurately describe chronic bronchitis? (Select 3) - higher risk of cor pulmonale - Loss of elastic recoil - Decreased airway diameter - Normal PaCO2 - Polycythemia - Decreased diffusing capacity

- higher risk of cor pulmonale - Decreased airway diameter - Polycythemia

A Patient with cor pulmonale due to COPD presents for an inguinal hernia repair. All of the following signs are consistent with cor pulmonale except: - pulmonary hypertension - increased pulmonary artery occlusion pressure - hepatomegaly - lower extremity edema

- increased pulmonary artery occlusion pressure

Anesthesia-induced atelectasis is best reversed by: - increasing peak airway pressure to 30 cm H2O for 8 seconds - increasing peak airway pressure to 40 cm H2O for 8 seconds - using a tidal volume of 6 ml/kg and PEEP of 5 cm H2O - using a tidal volume of 6 ml/kg and PEEP of 10 cm H2O

- increasing peak airway pressure to 40 cm H2O for 8 seconds

All of the following are landmarks for Larson's maneuver, except for: - skull base - mandibular body - ramus of mandible - mastoid process

- mandibular body

All of the following are appropriate in the setting of an acute bronchspasm except: - montelukast - sevoflurane - albuterol - hydrocortisone

- montelukast

Topically applied ophthalmic medications are absorbed: - as quickly as intravenous administration - more quickly than subcutaneous administration - only minutely, with insignificant clinical effect - directly into the central nervous system through the optic nerve foramen

- more quickly than subcutaneous administration Topically applied ophthalmic medications are absorbed at a rate intermediate between intravenous and subcutaneous injection. Children and the elderly are at particular risk for the toxic effects of topically applied medications.

What muscle abducts the vocal cords? -thyroepiglottic -lateral circoarytenoid -circothyroid -posterior cricoarytenoid

- posterior cricoarytenoid

What is the NPO policy for the following liquids? Use the ASA Practice Guidelines. 1. Breast Milk 2. Apple Juice 3. Cow's milk A. 6 Hours B. 4 Hours C. 2 Hours

1. --> B 2. --> C 3. --> A

The rate of seroconversion after exposure of mucous membranes to HIV-infected blood is approximately: 0.03% 0.09% 0.3% 0.9%

0.09% - Percutaneous exposure (needle stick) carries a risk of HIV-seroconversion of approximately 0.3% or about 1:300. Mucous membrane exposure carries a risk of approximately 0.09% or about 1:1100.

How much oxygen is consumed by a 70 kg healthy adult at rest? (Enter your answer as mL/100g/min)

0.357 ml/100g/min *The classic answer for a 70kg adult is 250 mL/min *The classic answer on a per weight basis is 3.5 ml/kg/min ***Commit both of these to memory

Match each concept with it's definition: 1. Haldane Effect 2. Hamburger Phenomenon 3. Bohr Effect A. Cl- is exchanged for HCO3- to maintain electroneutrality in the erythrocytes B. Hgb releases for O2 in an acidic environment C. Venous blood carries more CO2 than arterial blood

1. --> C 2. --> A 3. --> B

Match the upper airway dilator muscle to its function 1. Tensor palatine 2. Genioglossus 3. Hyoid muscles A. Opens the hypophayrnx B. Opens the oropharynx C. Opens the nasopharynx

1 > C 2 > B 3 > A

Match the nerve to the structure that it innervates. 1. Superior Laryngeal 2. Recurrent Laryngeal 3. Trigeminal 4. Glossopharyngeal A. Trachea B. Anterior Tongue C. Vallecula D. Posterior Epiglottis

1. Superior Laryngeal > D. Posterior Epiglottis 2. Recurrent Laryngeal > A. Trachea 3. Trigeminal > B. Anterior Tongue 4. Glossopharyngeal > C. Vallecula

For the patient in the sitting position, order the cartilages from superior to inferior Cricoid, Epiglottis, Corniculate, Arytenoid

1.Epiglottis 2. Corniculate 3. Arytenoid 4. Circoid

Portal hypertension is defined as sustained portal vein pressure greater than: 5 mm Hg 10 mm Hg 20 mm Hg 25 mm Hg

10 mm Hg Portal hypertension is defined as a sustained portal vein pressure of 10 mm Hg or greater. This leads to the formation of portal-systemic collateral venous channels.

The recommended maximum leakage current allowed in operating room equipment is: 5 μA 10 μA 1 mA 5 mA

10 μA - 10 μA has been established as the recommended maximum allowable leakage current. This amount of current is below the threshold of perception (1mA) as well as below the threshold for risk of microshock.

How many colony-forming units are required from a quantitative bronchoalveolar lavage (BAL) to make the diagnosis of pneumonia? 1,000 CFU/mL 10,000 CFU/mL 100,000 CFU/mL 1,000,000 CFU/mL

10,000 CFU/mL The cutoff point used for the diagnosis of pneumonia is 10,000 CFU/mL for BAL (for endotracheal aspirates the cutoff is 1,000,000 CFU/mL, and for a protected brush specimen the cutoff is 1,000 CFU/mL). Diagnosing a pneumonia with less than 10,000 CFU/mL based on a BAL runs the risk of inappropriately administering antibiotics, and theoretically runs the risk of increased antimicrobial resistance.

Intracranial hypertension is defined as a sustained increase in intracranial pressure (ICP) above: 5 mm Hg 15 mm Hg 25 mm Hg 30 mm Hg

15 mm Hg - Intracranial hypertension is defined as a sustained increase in intracranial pressure (ICP) above 15 mm Hg. Uncompensated increases in tissue or fluid within the rigid intracranial vault produce the sustained pressure elevations. - Longnecker pp. 871-874

At approximately what carboxyhemoglobin level will a patient start to display mild signs and symptoms (headache, nausea and vomiting) of carbon monoxide toxicity? 5% 10% 15% 25%

15% A nonsmoker has a carboxyhemoglobin level < 5% whereas a smoker may have a carboxyhemoglobin level of 4-9%. Symptoms such as headache, dizziness, nausea, and vomiting may occur at a carboxyhemoglobin level of 15-20%.

A 34-year-old female is undergoing tumescent liposuction of the abdomen and flanks. Peak serum levels of local anesthetic from the tumescent solution are most commonly seen in: the first 2 hours 4 to 6 hours 7 to 9 hours 12 to 14 hours

12 - 14 hours - Tumescent liposuction is commonly done with large volumes of tumescent solution consisting of normal saline with 1:1,000,000 epinephrine and 0.025 - 0.1% lidocaine. Peak serum levels of lidocaine occur 12 - 14 hours after injection and decline over the next 6 - 14 hours. - Barash pg 868

Which of the following BEST estimates the incidence of epidural spread following a lumbar plexus (psoas) block ? 0.1% 1% 16% 54%

16% Epidural spread following a lumbar plexus block is not uncommon. The incidence of epidural spread has been reported to range from 1.8%-16%.

A 56-year-old man is undergoing a right carotid endarterectomy with intraoperative EEG monitoring. Which of the following cerebral blood flow rates is associated with EEG deterioration? 10-15 mL/min/100 gm 15-20 mL/min/100 gm 20-25 mL/min/100 gm 25-30 mL/min/100 gm

15-20 mL/min/100 gm EEG provides evidence of electrical activity to cortical surface cells, which is an area vulnerable to ischemia. Regional cerebral blood flow is 50-55 mL/min/100gm brain tissue and ischemia begins at 18-20 mL/min/100gm and thus EEG deterioration becomes evident in the range of 15-20 mL/min/100gm. EEG signs of ischemia include slowing of the EEG in addition to amplitude reduction. In cases of severe ischemia, the EEG becomes isoelectric. There are some limitations to use of EEG to monitor neurologic status during carotid surgery and these include that deeper structures are not able to be monitored, preexisting deficits are not accounted for or may result in lack of intraoperative EEG changes. EEG changes are also affected by temperature, blood pressure, CO2 tension and anesthetic depth.

The loss of ventricular filling as a result of acute atrial fibrillation is approximately:

15-25% - Passive flow accounts for about 75 - 85% of ventricular filling. The remaining 15 - 25% occurs as a result of atrial contraction, which is lost during atrial fibrillation. - Barash pg. 249

Which type of pneumocyte produces surfactant? (Enter whole #)

2

The age group with the highest minimum alveolar concentration (MAC) of desflurane is: 2 - 3 months 1 - 2 years 25 - 30 years greater than 75 years

2 - 3 months - The two-to-three-months-of-age group represents the highest MAC requirement. MAC subsequently decreases with advancing age. - Morgan & Mikhail pg. 883

What is the best size LMA classic for a 7 year old that weighs 54 lbs?

2.5

During an anterior-posterior spinal fusion, in a 70-kg patient, the laboratory reports an intraoperative hematocrit of 21% with a hemoglobin of 7g/dL. Two units of packed red blood cells are administered over the course of 30 minutes. Upon completion of the transfusion, the anticipated hematocrit will be approximately: (Enter numerical answer in box below. Click 'Next' when completed.)

25-27 During an anterior-posterior spinal fusion, in a 70-kg patient, the laboratory reports an intraoperative hematocrit of 21% with a hemoglobin of 7g/dL. Two units of packed red blood cells are administered over the course of 30 minutes. Upon completion of the transfusion, the anticipated hematocrit will be approximately: (Enter numerical answer in box below. Click 'Next' when completed.) 25 - 27% A commonly used rule of thumb states that each unit of PRBCs increases the hemoglobin 1 g/dL and the hematocrit 2% to 3%.

The National Institute for Occupational Safety (NIOSH) recommends limiting the operating room concentration of nitrous oxide to: 0.5 ppm 5 ppm 25 ppm 50 ppm

25 ppm - NIOSH recommends limiting the room concentration of nitrous oxide to 25 ppm and halogenated agents to 2 ppm (0.5 ppm if nitrous oxide is also being used). - pp. 652-654 Longnecker, DE, Brown, DL, Newman MF and Zapol, WM. Anesthesiology. New York: McGraw Hill, 2012.

How many unpaired cartilages are present in the Larynx? (Enter whole #)

3

Actuation of the oxygen flush valve delivers 100% oxygen at a rate of: 10 - 20 L/min 20 - 30 L/min 35 - 75 L/min 80 - 100 L/min

35 - 75 L/min The oxygen flush valve delivers 100% oxygen at a rate of 35 - 75 L/min with a pressure of 40 - 60 psi.

The end-tidal CO2 is 5%. Convert this to mmHg (Assume you are at sea level and enter your answer as a whole number)

38 mmHg *Calculate 5% of atmospheric pressure at sea level (760 mmHg)

A baby is born at 39 weeks gestation with congenital diaphragmatic hernia. The baby is cyanotic and limp. He has a slow, irregular respiratory pattern and appears to be grimacing. His heart rate is 120 bpm at the one-minute of life. Which APGAR score correctly corresponds with these findings? 3 4 5 6

4 APGAR stands for Appearance, Pulse, Grimace, Activity, and Respirations. Each of the 5 signs is assigned a value from 0 to 2 with a total greater than 7 considered normal. In this question, the baby is cyanotic with a pulse greater than 100, he is grimacing, he has no tone, and he has irregular respirations.

What is the largest size ETT that an be passed through a LMA classic 2

4.5

A 24-year-old female is scheduled for resection of a cerebral aneurysm. She has no other significant past medical history. Acceptable levels of hypotension would include a mean arterial pressure of: 20 - 30 mm Hg 35 - 45 mm Hg 50 - 60 mm Hg 90 - 100 mm Hg

50 - 60 mm Hg Healthy young individuals tolerate mean arterial pressures as low as 50 - 60 mm Hg without complications. Chronically hypertensive patients have altered autoregulation of cerebral blood flow and may tolerate a mean arterial pressure of no more than 20 - 30% below baseline.

During emergent transtracheal jet ventilation using a 14 gauge catheter, generation of sufficient gas flow requires a driving pressure of: 20 cmH2O 50 cmH2O 25 psi 50 psi

50 psi - During emergent transtracheal jet ventilation using a 14 gauge catheter, generation of sufficient gas flow requires a driving pressure of: 50 psi - After proper location of the catheter is confirmed by aspiration air, jet ventilation may be achieved with intermittent pulses of oxygen at 50 psi. - Morgan & Mikhail's Clinical Anesthesiology. - pg. 1237

During emergent transtracheal jet ventilation using a 14 gauge catheter, generation of sufficient gas flow requires a driving pressure of: 20 cmH2O 50 cmH2O 25 psi 50 psi

50 psi: During emergent transtracheal jet ventilation using a 14 gauge catheter, generation of sufficient gas flow requires a driving pressure of: After proper location of the catheter is confirmed by aspiration air, jet ventilation may be achieved with intermittent pulses of oxygen at 50 psi.

In the fetus, the percentage of cardiac output directed to the placenta is approximately: 10% 25% 50% 100%

50% In the fetus, the lungs receive little blood flow. The placenta receives nearly one-half of the fetal cardiac output and is responsible for respiratory gas exchange.

The cuff pressure for a LMA classic should not exceed (Enter answer in cm H20)

60 cm H2O

At what age does the clearance of morphine in pediatric patients reach adult values? 1 month 6 months 18 months 24 months

6 months Morphine is metabolized by uridine 5′‐diphosphate glucuronosyltransferase (UGT) to morphine‐3‐glucuronide (M3G) and morphine‐6‐glucuronide (M6G). The clearance and elimination of morphine is age-dependent. The clearance of morphine is reduced during the neonatal period and increases with increasing age but there is significant inter-individual variability. Most studies suggest total body morphine clearance is 80% that of adult values by 6 months of age. Children at 18 months and 24 months are already clearing morphine at the adult level. In fact, there is some evidence that they clear morphine at a greater rate than adults.

A 42-year-old man is undergoing a thoracoscopy. During the procedure an 8-minute period of apneic oxygenation is required. If the patient's PaCO2 is 40 mm Hg, the expected PaCO2 at the end of the apneic period would be: ____mmHg

67 to 74 mm Hg The apneic oxygenation technique affords adequate oxygen delivery, but progressive respiratory acidosis limits the use of this technique to 10 - 20 minutes in most patients. Arterial PaCO2 rises 6 mm Hg in the first minute followed by a rise of 3 - 4 mm Hg during each subsequent minute. In this patient this will produce a 27 - 34 mm Hg increase, resulting in a PaCO2 of 67 to 74 mm Hg.

A full-term, 4.2 kg neonate is scheduled for a thoracotomy for resection of congenital lobar emphysema. The infant's starting hematocrit is 48%. Estimated allowable blood loss to maintain a hematocrit at or above 38% is: (Enter numerical answer)

70 - 110 mL The full-term neonate has approximately 85 ml/kg total blood volume. Therefore: 4.2 kg x 85 ml/kg = 357 ml (blood volume) MABL = Blood Volume x (HCT(starting) - HCT(final)) / HCT(average) 357 ml x (48 - 38) / 43 = 83 mL

A full-term, 4.2 kg neonate is scheduled for a thoracotomy for resection of congenital lobar emphysema. The infant's starting hematocrit is 48%. Estimated allowable blood loss to maintain a hematocrit at or above 38% is:

70 - 110 mL The full-term neonate has approximately 85 ml/kg total blood volume. Therefore: 4.2 kg x 85 ml/kg = 357 ml (blood volume) MABL = Blood Volume x (HCT(starting) - HCT(final)) / HCT(average) 357 ml x (48 - 38) / 43 = 83 mL

During the delivery of an anesthetic in the radiology department, full E-cylinders of nitrous oxide and oxygen are being used. If a 3:2 mixture of nitrous oxide:oxygen is being delivered and the case has been proceeding for 60 minutes, the expected pressure in the nitrous oxide E-cylinder is: _______

745 - 750 psig - Nitrous oxide has a critical temperature of 37oC. This allow nitrous oxide to exist as a liquid at room temperature. Full E-cylinders of nitrous oxide contain approximately 1590 L at a pressure of 745 psig. A sixty minute delivery of 3 L/min would result in a 180 L consumption, and this would be inadequate to consume all the liquid nitrous oxide in the tank. As a result, there would be no change in tank pressure.

During the delivery of an anesthetic in the radiology department, full E-cylinders of nitrous oxide and oxygen are being used. If a 3:2 mixture of nitrous oxide:oxygen is being delivered and the case has been proceeding for 60 minutes, the expected pressure in the nitrous oxide E-cylinder is: ___ psig

745 - 750 psig During the delivery of an anesthetic in the radiology department, full E-cylinders of nitrous oxide and oxygen are being used. If a 3:2 mixture of nitrous oxide:oxygen is being delivered and the case has been proceeding for 60 minutes, the expected pressure in the nitrous oxide E-cylinder is: 745 - 750 psig Nitrous oxide has a critical temperature of 37oC. This allow nitrous oxide to exist as a liquid at room temperature. Full E-cylinders of nitrous oxide contain approximately 1590 L at a pressure of 745 psig. A sixty minute delivery of 3 L/min would result in a 180 L consumption, and this would be inadequate to consume all the liquid nitrous oxide in the tank. As a result, there would be no change in tank pressure.

During the delivery of an anesthetic in the radiology department, full E-cylinders of nitrous oxide and oxygen are being used. If a 3:2 mixture of nitrous oxide:oxygen is being delivered and the case has been proceeding for 60 minutes, the expected pressure in the nitrous oxide E-cylinder is: _____ psig

745 - 750 psig Nitrous oxide has a critical temperature of 37oC. This allows nitrous oxide to exist as a liquid at room temperature. Full E-cylinders of nitrous oxide contain approximately 1590 L at a pressure of 745 psig. A sixty minute delivery of 3 L/min would result in a 180 L consumption, and this would be inadequate to consume all the liquid nitrous oxide in the tank. As a result, there would be no change in tank pressure.

A 67-year-old man with no prior heparin exposure underwent an uneventful aortic valve replacement. Three days after surgery, the patient has an isolated and asymptomatic drop in platelet count > 50%. What is the probability of heparin-induced thrombocytopenia? < 1% 1-5% 5-10% > 10%

< 1% Thrombocytopenia is common after cardiac surgery and is typically multifactorial. The most feared cause is heparin-inducted thrombocytopenia (HIT). HIT is an immune-mediated reaction involving antibodies to a heparin: PF4 complex (PF4 is a cytokine released from alpha granules of activated platelets during activation, and its physiologic role is to neutralize heparin-like molecules in the extracellular matrix). This can lead to platelet clumping and a prothrombotic state. The treatment for HIT is anticoagulation with a direct thrombin inhibitor (DTI). Warfarin is contraindicated in the setting of HIT because it can lead to coagulation ("Coumadin necrosis"). Suspicion of HIT complicates venous thromboprophylaxis because if HIT is present, heparin and low molecular weight heparin should be avoided. Thus, a four-point scoring system was developed based on the amount of thrombosis, the timing, alternative causes, and physical exam findings. In a patient with plausible other causes (i.e.., cardiopulmonary bypass) who is asymptomatic and outside the 5-10 day peak period, an isolated platelet drop of 50% or more still only carries a 0.84% chance of HIT.

The body mass index (BMI) associated with morbid obesity is: > 30 > 35 > 40 > 45

> 40 Overweight and obesity are classified using the BMI. Overweight is defined as a BMI > 24, obesity as a BMI > 30, morbid obesity as a BMI > 40, super obesity as a BMI > 50 and super-super obesity as a BMI > 60. BMI is calculated with the following formula: BMI = Weight (kg) / Height (meters)2

An airway fire has occurred. What are the next steps (order from first to last) A. Remove ETT B. Pour saline into airway C. Stop flow of all airway gases D. Re-establish ventilation

A. C. B. D.

The most severe transfusion reactions are due to: ABO incompatibility Rh incompatibility febrile reactions non-ABO hemolytic reactions

ABO incompatibility - The most severe transfusion reactions are due to ABO incompatibility. Naturally acquired antibodies can react against the transfused antigens, activate complement and result in intravascular hemolysis. - M&M pg 1172

A 9-month-old infant presents to the OR for a right inguinal hernia repair. The newborn screening was positive for beta-thalassemia major. Which of the following MOST accurately defines this condition? Absence of one of the beta-globin proteins Absence of both of the beta-globin protein Absence of one of the alpha-globin proteins Absence of both of the alpha-globin proteins

Absence of both of the beta-globin protein Normal hemoglobin has two alpha and two beta globin protein chains. A deficiency or absence of the alpha chain results in Alpha-thalassemia and a deficiency or absence of one or more of the beta chains results in Beta-thalassemia. The beta chain requires a gene from each parent to produce the correct quality and quantity of beta globin. If the gene from one parent is missing or incomplete, the child will develop Beta-thalassemia minor. If the gene from both parents is missing, the child will develop Beta-thalassemia major or Cooley's anemia.

Which of the following neurotransmitters is MOST commonly used by preganglionic sympathetic neurons at sympathetic ganglia? Acetylcholine Dopamine Epinephrine Norepinephrine

Acetylcholine Both sympathetic and parasympathetic PREganglionic neurons are cholinergic and thus release acetylcholine as the neurotransmitter. Most POSTganglionic sympathetic neurons are adrenergic and release norepinephrine; however, some are cholinergic such as the nerves that innervate sweat glands. Most POSTganglionic parasympathetic neurons are and thus release acetylcholine.

A 44-year-old man presents to the emergency room with a table saw injury to the left hand. The surgeon plans to explore the wound and possibly repair tendon and vascular injuries. The estimated operative duration is between 1-5 hours and an axilla block with bupivacaine is planned as the primary anesthetic technique. The patient has a seizure with the inejction of bupivacaine. Which of the following should be your FIRST therapeutic action? Administer a short-acting muscle relaxant to stop muscle contractions Administer intra-lipid Administer lidocaine to prevent cardiac arrhythmias Administer oxygen and ensure a patent airway

Administer oxygen and ensure a patent airway The treatment of local anesthetic toxicity is similar to the management of other medical emergencies and focuses on airway, breathing and circulation. Ensuring adequate oxygenation and ventilation is paramount to avoid progressive acidosis.

Which of the following is NOT a potential treatment for salicylate poisoning? Activated charcoal Administration of a reversible COX-inhibitor Hemodialysis Sodium bicarbonate

Administration of a reversible COX-inhibitor

A 55-year-old woman with a history of congenital long QT syndrome is undergoing a hysteroscopy for abnormal uterine bleeding. She had uneventful induction of general anesthesia but after paracervical block with lidocaine develops ventricular tachycardia with morphological appearance of torsades de pointe. Which of the following medications should be AVOIDED in the treatment of her arrhythmia? Amiodarone Calcium chloride Esmolol Magnesium sulfate

Amiodarone Congenital long QT syndrome may occur in conjunction with other hereditary syndromes, such as Jervell, Lange-Nielsen or Romano-Ward syndrome, or acquired as a result of pharmacologic or metabolic etiologies. It is an issue of cellular repolarization which precipitates tachyarrhythmias, most commonly polymorphic ventricular tachycardia or torsades de pointe. There are multiple subtypes that affect both potassium and/or sodium channels. The arrhythmias may be precipitated by sympathetic activation, auditory stimuli or at rest. Family history may be positive for sudden cardiac death and the ECG significant for prolonged corrected QT interval > 430ms or bizarre odd-appearing T waves. Treatment includes magnesium for arrhythmias, possible permanent pacemaker, or beta blockers for subtypes 1 and 2, but amiodarone is considered contraindicated as it prolongs the QT interval.

A previously healthy 32-year-old G1P0 patient at 39 weeks gestation is rushed to the operating room for cesarean delivery after fetal heart rate tracings display prolonged late decelerations. The patient is induced with propofol, intubated by rapid sequence with succinylcholine, and maintained with 1.8% isoflurane in 100% oxygen. The fetus is delivered safely, but the patient's blood pressure is noted to be 85/40 mmHg throughout the procedure, down from a baseline of 115/60 mmHg; it responds appropriately and transiently to ephedrine and phenylephrine boluses. The heart rate range is 70-90 bpm, SpO2 is 100% and blood loss is estimated at 900 mL. What is the most likely etiology of hypotension? Anesthetic overdose Amniotic fluid embolism Chorioamnionitis Postpartum hemorrhage

Anesthetic overdose Pregnant patients have decreased MAC requirements of 25-40% compared to non-pregnant women. Isoflurane at 1.8% would be a high dose even in non-pregnant patients (roughly 1.5 MAC), but adjusting for pregnancy it would be closer to 1.7- 2.0 MAC, or approximately the level at which the sympathetic system stops responding to stimuli (MAC-bar). Such hypotension should respond well to pharmacologic sympathetic stimulation with ephedrine or phenylephrine. In contrast, amniotic fluid embolism rapidly deteriorates to total cardiovascular collapse. The blood loss is typical for a cesarean delivery and since the heart rate is not increased, postpartum hemorrhage is less likely. Chorioamnionitis would likely have been present prior to induction of anesthesia, and associated with fever, tachycardia, and possibly hypotension prior to arrival in the operating room.

After a CABG, a 65-year-old man is hypotensive upon weaning from cardiopulmonary bypass. The surgeon is considering insertion of an intraaortic balloon pump for counterpulsation. Which of the following represents the MOST significant contraindication to placement of an intraaortic balloon pump? Aneurysm of left ventricle Aortic atheroma Aortic regurgitation Aortic stenosis

Aortic regurgitation Intraaortic balloon counterpulsation has been used for many years for patients with left ventricular failure. Indications for its use have grown in recent years. Still, it is most commonly used for patients in cardiogenic shock, post-myocardial infarction, severe myocarditis, cardiomyopathy, unstable angina refractory to medications, high grade left main coronary artery disease, failure to wean adequately from cardiopulmonary bypass, low cardiac output syndrome and as a short-term bridge to heart transplant. There are relatively few contraindications, including severe aortic regurgitation (AR) as the balloon inflates during diastole and would worsen AR and not improve coronary flow during diastole. Aortic dissection (difficult to place balloon in true lumen of aorta) and severe aortoiliac occlusive disease or peripheral vascular disease (high risk of limb ischemia) are also contraindications.

In which of the following cardiac conditions would a patient presenting in shock be LEAST likely to benefit from intra-aortic balloon pump (IABP) placement? A ruptured papillary muscle from a recent myocardial infarction A low ejection fraction of 10% secondary to ischemic cardiomyopathy Aortic valve endocarditis resulting in severe aortic insufficiency Severe aortic stenosis and congestive heart failure

Aortic valve endocarditis resulting in severe aortic insufficiency A patient with severe aortic insufficiency should not have an IABP placed since it would worsen the amount of blood regurgitated into the left ventricle and likely lead to left ventricular distension. Since the IABP improves coronary perfusion pressure and unloads the left ventricle, the other conditions would benefit from IABP placement.

An 18-year-old man presents with pinpoint pupils, salivation, confusion, GI upset, and impending respiratory failure after performing yard work for a neighbor. Which of the following medications is MOST appropriate? Atropine Echothiophate Glycopyrrolate Physostigmine

Atropine Organophosphate compounds are found in some insecticides and can also be used in chemical warfare. They form an irreversible bond with acetylcholinesterase causing its inhibition and an accumulation of acetylcholine at cholinergic junctions. Atropine is a competitive muscarinic receptor antagonist. Atropine must be administered in cases of organophosphate toxicity to mitigate impending respiratory failure and other side effects of acetylcholine at the muscarinic receptor. Importantly, atropine crosses the blood brain barrier unlike glycopyrrolate, thereby affecting centrally mediated respiratory depression. It also serves as an antisialagogue to dry copious pulmonary secretions. It is important to note that atropine cannot reverse the organophosphate toxicity. Thus, the antidote pralidoxime should also be administered. Pralidoxime, or 2-pyridine aldoxime methyl chloride (2-PAM), is an agent that reactivates acetylcholinesterase if the irreversible binding of the organophosphate compounds to acetylcholinesterase has yet to occur, typically within 48 hours of poisoning. Pralidoxime serves to reverse the nicotinic muscle paralysis associated with organophosphate toxicity.

You are evaluating a 4-year-old boy for an urgent endoscopy. He was previously healthy and active but complained of a severe sore throat earlier today. His family elected to avoid vaccines for fear of autism. On exam, he is febrile and is sitting upright with his hands and on his knees with his neck extended. He is tachypneic and has excess salivation from is mouth. His voice is muffled and you note inspiratory stridor. Which of the following is the MOST likely explanation for his respiratory distress? Bacterial epiglottitis Foreign body Peritonsillar abscess Viral Croup

Bacterial epiglottitis Epiglottitis is an uncommon bacterial infection of the epiglottis. It is usually rapid onset and presents with high fever, leukocytosis, and significant respiratory distress. Patients may be sitting upright, salivating, with their head extended. Haemophilus influenzae used to be a common cause of epiglottis but this has diminished significantly with the Haemophilus influenzae type B vaccine.

Basal metabolic oxygen consumption in a 20-kg patient is approximately: (Enter numerical answer in box below. Click 'Next' when completed.) mL/min

Basal metabolic oxygen consumption in a 20 kg patient is approximately: 95 ml/min Basal metabolic oxygen consumption can be estimated using the following formula: VO2 = 10kg^3/4

Which of the following sequence of events MOST commonly occurs in drowning? Breath-holding, laryngospasm, involuntary ventilatory efforts, desaturation, muscular relaxation, inhalation of water, cardiac arrest Breath-holding, involuntary ventilatory efforts, inhalation of water, laryngospasm, desaturation, muscular relaxation, cardiac arrest Involuntary ventilatory efforts, laryngospasm, breath-holding, desaturation, muscular relaxation, inhalation of water, cardiac arrest Breath-holding, laryngospasm, involuntary ventilatory efforts, muscular relaxation, inhalation of water, desaturation, cardiac arrest

Breath-holding, laryngospasm, involuntary ventilatory efforts, desaturation, muscular relaxation, inhalation of water, cardiac arrest Drowning is defined as a primary respiratory impairment from submersion or immersion in a liquid medium. The process of drowning usually entails a voluntary period of breath-holding (for an average of 87 seconds), possible (but not always) swallowing of water, followed by laryngospasm as water becomes entrained into the larynx. Involuntary ventilatory efforts ensue but are ineffective in the setting of laryngospasm. Survivors report this as being the most traumatic component of the drowning process and the total duration of voluntary and involuntary breath holding lasts 1.5-2 minutes. As arterial saturation declines, laryngospasm eventually abates, at which point the subject begins to breathe in water, after which circulatory arrest quickly occurs. The organ most susceptible to injury during drowning is the brain, with brain damage occurring approximately 3 minutes after PaO2 falls below 30 mm Hg. Exceptions to this rule include trained divers and individuals immersed in cold water. Many subjects who drown do so under direct supervision (lifeguards falsely believe that the victims were "fooling around" or attend to other tasks). No individual should ever be allowed under water without making purposeful movements for more than 10 seconds. Treatment: airway management is of paramount importance and should be initiated in water if a) this is safe and b) removal from water would result in any time delay. Once ventilation has been initiated, attend to the cardiovascular system - not all patients will have systole and profound bradycardia is not uncommon. Abdominal thrusts to "remove water" have not been shown to be helpful. The major sources of morbidity and mortality are related to respiratory and CNS compromise. Post-drowning survivors should be admitted to an ICU and CPAP (or PEEP) applied. Anti-epileptics may be initiated in patients with suspected CNS injury. Infection may be an issue in standing water (Pseudomonas) and empiric antibiotic coverage may be indicated in this setting.

What is the MOST likely position of the larynx in a normal full term infant? C2-C3 C3-C4 C4-C5 C5-C6

C3-C4 The position of the larynx in the full term infant is C3-C4. The position in the preterm neonate is even more cephalad at C3. The position in the adult is C4-C5.

You are caring for a neonate with congenital diaphragmatic hernia and pulmonary hypertension. She is intubated and receiving inhaled nitric oxide (iNO). iNO reduces pulmonary vascular resistance by activating protein kinase G via cyclic GMP. This reduces which of the following intracellular ions? Calcium Magnesium Potassium Sodium

Calcium iNO activates guanylate cyclase. This in turn converts GTP to cGMP. cGMP activates protein kinase G which decreases intracellular calcium by decreasing Ca++ entry into the muscle cell and by decreasing the release of Ca++ from the sarcoplasmic reticulum. The reduced concentration of calcium decreases Ca++ activated phosphorylation of myosin.

A patient with a creatinine of 1.6 mg/dL is receiving IV magnesium for preeclampsia. The patient develops a widened QRS complex. After cessation of magnesium administration, which of the following is the MOST appropriate therapy? Calcium chloride Glucagon Kayexalate Propranolol

Calcium chloride This preeclamptic patient, who was receiving magnesium for seizure prophylaxis, presumably developed worsening renal function as a complication of her preeclampsia. Since magnesium is cleared renally, her serum magnesium levels rapidly increased, leading to magnesium toxicity. Normal serum levels of magnesium are 1.2-2.0 mEq/L and therapeutic range is 4-8 mEq/L. ECG changes including prolonged PR intervals and widened QRS complexes can develop at serum levels of 5-10 mEq/L, with loss of deep tendon reflexes being another telltale sign of magnesium toxicity. Calcium chloride (500mg) or calcium gluconate (1g) are the antidotes for magnesium toxicity, via competitive inhibition of divalent cations. Miller RD, Pardo M. Basics of Anesthesia, 6th ed. Philadelphia, PA: Elsevier Saunders; 2011.

A 57-year-old man with ESRD develops progressive bradycardia with peaked T-waves following reperfusion of the renal allograft during a kidney transplant. Which of the following is the MOST appropriate initial treatment? Calcium chloride 500-1000 mg IV Glucose 25-50g/Insulin 5-10 U IV Hyperventilation to PaCO2 30 mmHg Sodium bicarbonate 8.4 % 50 ml IV

Calcium chloride 500-1000 mg IV This patient has ECG changes that suggest symptomatic hyperkalemia. All the options listed would effectively lower serum potassium. Glucose/Insulin, hyperventilation, and bicarbonate all function by shifting potassium from the extracellular space into the intracellular space. Unfortunately, each of these interventions takes time (5-10 minute minimum) to be effective. Calcium is a physiologic antagonist and can temporarily stabilize the myocardium. Calcium is effective almost immediately and thus is the initial treatment of choice in this patient.

In planning for the intubation of the full-term parturient patient who requires general anesthesia, which of the following is MOST correct? - Enlarged tracheal caliber due to changes in cartilage composition at term may necessitate selected of a larger ETT - Capillary engorgement of mucosal lining of the upper airway, including vocal cords and arytenoids, leads to tissue edema and may necessitate selection of smaller ETT - Pre-oxygenation for a full one minute should precede induction of general anesthesia - Anesthetic requirements may be greater

Capillary engorgement of mucosal lining of the upper airway, including vocal cords and arytenoids, leads to tissue There are many changes of the upper airway during pregnancy. Capillary engorgement in the mucosal lining of the upper respiratory tract leads to tissue edema in normal full-term pregnant patient resulting in smaller caliber upper airway. Edema at the level of the arytenoids and vocal cords may necessitate selection of a smaller ETT (6.0 or 6.5mm). Additionally, increased tissue friability can be expected and extra care should be taken during any instrumentation of the mouth or upper airway, such as suctioning, placement of an oral airway or direct laryngoscopy. Active pushing leading to venous engorgement can exacerbate tissue edema along with the above-described changes. Given the decrease in functional residual capacity and increased oxygen consumption during pregnancy, pre-oxygenation for at least three minutes is recommended prior to induction of general anesthesia. Anesthetic requirements are decreased in pregnancy and thus MAC is decreased.

A 55-year-old previously healthy woman presents with trigeminal neuralgia. Which of the following agents is the MOST appropriate first-line pharmacologic treatment? Amitriptyline Carbamazepine Oxycodone Phenytoin

Carbamazepine Trigeminal neuralgia is characterized by intense, stabbing pain in the distribution of the trigeminal nerve. The symptoms are usually unilateral. The patient may experience exacerbations with more frequent attacks, followed by remissions with fewer and less frequent attacks. Carbamazepine is the first line treatment for trigeminal neuralgia.

A 2-day-old boy with a lumbar meningomyelocele is scheduled to undergo surgical repair. Which of the following associated anomalies are MOST likely to occur with a meningomyelocele? Cleft palate Chiari malformation Radial anomalies Renal failure

Chiari malformation Infants with meningomyelocele have many associated anomalies. The most common associated anomalies include Chiari malformations. There are 4 types of Chiari malformations. The result of the Chiari malformation is downward displacement of the brainstem, cerebellar tonsils and fourth ventricle. This causes hydrocephalus, which usually requires surgical shunting. Patients can present with apnea and stridor.

Which of the following local anesthetics has the LEAST amount of maternal-fetal transfer across the placenta? Bupivacaine 0.25% Chloroprocaine 3% Lidocaine 1.5% Ropivacaine 0.2%

Chloroprocaine 3% Transfer of drugs across the placenta is governed by the Fick principle, which states that rate of placental transfer of a drug is DIRECTLY proportional to (1) surface area available for exchange, (2) concentration gradient between maternal and fetal blood and INVERSELY proportional to the thickness of the membrane across which transfer takes place. Chloroprocaine is unique among the local anesthetics listed above in that it is metabolized by plasma esterases, which decreases the maternal plasma concentration and thus lowers the maternal-fetal concentration gradient. Thus, very little if any chloroprocaine can be expected to be transferred to the fetus.

Which of the following statements regarding the addition of clonidine to an epidural infusion is MOST correct? - Clonidine is an alpha-2-receptor antagonist - Clonidine effect in peripheral nerve blocks is primarily centrally mediated - Clonidine prolongs the sensory block when added to intermediate local anesthetics - Commonly reported side effects include tachycardia and hypertension

Clonidine prolongs the sensory block when added to intermediate local anesthetics Clonidine is an alpha-2-agonist. It has been found to prolong the sensory block primarily when combined with intermediate local anesthetics although it may also have some effect when combined with longer-acting local anesthetics. The exact mechanism is unclear but appears to be peripherally mediated and dose-dependent. Side effects include bradycardia, hypotension, and sedation.

Which of the following anti-platelet agents works by inhibiting platelet activation via an adenosine diphosphate-dependent mechanism? Aspirin Clopidogrel Dipyridamole Tirofiban

Clopidogrel Aspirin permanently inactivates the cyclooxygenase enzyme to reduce thromboxane A2-induced activation of platelets. Clopidogrel is one of the thienopyridines which functions via irreversibly inhibiting ADP-induced platelet aggregation. Other medications in this class include prasugrel, ticlopidine (both irreversible) and cangrelor and ticagrelor (both reversible). Dipyridamole is a phosphodiesterase inhibitor which achieves its antiplatelet effect through increased cyclic AMP which blocks the uptake of adenosine, thus reducing the amount of adenosine at the platelet vascular interface or via direct stimulation of prostacyclin release from the endothelium. Tirofiban is in the class of antiplatelet agents which block platelet activation via glycoprotein IIb/IIIa inhibition.

Before going back to the OR for a hernia repair, a 75-year-old man with hypertension complains of chest pain. During your evaluation, he grabs his chest and falls to the floor unresponsive. He has no pulse and is not breathing. When providing chest compressions you should do which of the following? Provide compressions at least 90 times per minute Compress to a depth of 2 inches Rotate with relief person only when you begin to feel exhausted Keep EtCO2 above 20mmHg when endotracheal tube is placed

Compress to a depth of 2 inches ACLS recommends the following for high-quality chest compressions /n (1) pressing at a depth of at least 2 inches; /n (2) pressing at a rate of at least 100 compressions per minute; /n (3) rotating with relief person every 2 minutes; /n (4) minimizing interruptions in compressions; /n (5) avoiding excessive ventilation; /n (6) providing compressions to EtCO2 of 10mmHg and/or arterial pressure of 20mmHg if monitoring device is available.

Which of the following is LEAST likely to be a risk factor for the development of postherpetic neuralgia? Age > 60 years Corticosteroid administration Female sex Severe acute pain

Corticosteroid administration Advanced age has consistently been demonstrated to be associated with an increased risk of postherpetic neuralgia. Postherpetic neuralgia is more frequently observed in women and Jung et al. found female sex to be strongly associated with postherpetic neuralgia. Patients with more severe acute pain have an increased risk to develop postherpetic neuralgia. Corticosteroids may be added to antiviral therapy during the acute phase and may reduce acute pain; however, the impact of corticosteroids on the incidence of postherpetic neuralgia remains uncertain.

Regarding motor innervation of the larynx, which of the following muscles is NOT innervated by the recurrent laryngeal nerve? Cricothyroid muscle Lateral cricoarytenoid muscle Thyroarytenoid muscle Posterior cricoarytenoid muscle

Cricothyroid muscle The cricothyroid muscle is innervated by the external branch of the superior laryngeal nerve. The superior laryngeal nerve is a branch of the vagus nerve. The cricothyroid muscle is the only muscle of the larynx not innervated by the recurrent laryngeal nerve. The cricothyroid muscle is the only tensor muscle of the larynx; thus, it serves to aid phonation by increasing the pitch of the voice. The cricothyroid muscle pulls the anterior aspects of the thyroid and cricoid cartilages together to achieve this. The thyroarytenoid muscle, innervated by the recurrent laryngeal nerve, antagonizes this movement.

Which of the following is MOST often associated with carbon monoxide poisoning? Salicylate poisoning Cyanide toxicity Ethanol toxicity Acetaminophen toxicity

Cyanide toxicity Carbon monoxide binds to hemoglobin and prevents the delivery of O2 to tissues. Conventional two-wavelength pulse oximeter cannot discriminate between Hgb-CO and HgbO2. The half-life of carboxyhemoglobin (Hb-CO) in a patient breathing room air is approximately 300 minutes; this decreases to 90 minutes with high-flow oxygen via a nonrebreathing mask. Thus, the most important interventions in the management of a CO-poisoned patient are prompt removal from the source of CO and institution of high-flow oxygen by facemask. For patients suffering from CO poisoning after smoke inhalation, it is important to consider concomitant cyanide toxicity, which can further impair tissue oxygen utilization and exacerbate cellular hypoxia. Cyanide toxicity should be considered in anyone who rapidly loses consciousness after ingestion or inhalation (it is common in indoor fire victims due to combustion of plastics). The symptoms are those of hypoxia (headache, lethargy, seizures, coma but without cyanosis). Cyanide inhibits cytochrome oxidase in the electron transport chain which prevents the offloading of electrons from NADH to oxygen which prevents ATP production → anaerobic metabolism → lactic acidosis all in spite of normal PaO2. Manage with 100% O2 and give any or all of amyl nitrate, sodium nitrate, and/or sodium thiosulfate. The nitrates generate methemoglobin which has a greater affinity for cyanide than does cytochrome oxidase.

What are the next steps after the tip of a double lumen ETT is placed between the vocal cords? (Order from 1st to last) A. Confirm placement with FOB B. Advance DTL into the bronchus C. Rotate the DLT 90 degrees D. Remove the stylet

D. C. B. A.

Local anesthetics with the potential to form methemoglobin include: (Select 3) EMLA topical anesthetic cream bupivacaine benzocaine ropivacaine prilocaine mepivacaine

EMLA topical anesthetic cream, prilocaine, benzocaine -EMLA cream contains both lidocaine and prilocaine. The metabolites of prilocaine can convert hemoglobin to methemoglobin. Benzocaine can also cause methemoglobinemia. -Nagelhout pg. 140

As compared with plasma osmolality, hypertonic crystalloid solutions include: D5W Ringer's lactate D5 0.25NS D5 0.45 NS

D5 0.45NS Normal plasma osmolality ranges between 280 - 290 mOsm/L. D5W is hypotonic in relation to plasma, with a tonicity of 253 mOsm/L. Both Ringer's lactate and D5 0.25NS are isotonic solutions, with tonicities of 273 and 355 mOSm /L respectively. D5 0.45NS is hypertonic with a tonicity of 406 - 432 mOsm/L.

A 45-year-old woman with a history of hypertrophic cardiomyopathy is undergoing a total abdominal hysterectomy for fibroids. Which of the following is MOST consistent with reducing the pressure gradient across the left ventricular outflow tract? Decrease afterload Decrease heart rate Increase contractility Reduce preload

Decrease heart rate Hypertrophic cardiomyopathy involves concentric ventricular hypertrophy along with significant, out-of-proportion septal hypertrophy. When the septum bulges into the left ventricular outflow tract during systole, with the hyperdynamic left ventricular contraction, the anterior leaflet of the mitral valve gets pulled down and a venturi effect is created, which causes a gradient across the left ventricular outflow tract. Anything that decreases the cavity size of the left ventricle worsens this outflow obstruction and thus it is said to be a dynamic obstruction. The goals for reducing the gradient include avoiding decreases in afterload and preload as well as avoiding tachycardia or increases in contractility.

Which of the following MOST accurately reflects changes in lung volumes during pregnancy? Decrease in functional residual capacity (FRC) Decrease in vital capacity (VC) Increased expiratory reserve volume (ERV) Increased residual volume (RV)

Decrease in functional residual capacity (FRC) During pregnancy, enlargement of the uterus forces the diaphragm cephalad, resulting in decreased residual volume and expiratory reserve volume. The decrease in RV and ERV create an approximately 20% decrease in FRC at term. Vital capacity does not change during pregnancy. Total lung capacity may decrease by as much as 5%.

You are called to the NICU to perform an anesthetic on a 3.5 kg neonate with congenital diaphragmatic hernia. The patient is intubated with conventional mechanical ventilation. The most recent arterial blood gas reveals a pH 7.38, PaCO2 45 mmHg, PaO2 89 mmHg, HCO3 29 mEq/L, SaO2 of 97% on an FiO2 of 0.6. Peak airway pressures are 32 cmH2O with 5 cmH2O PEEP. Expiratory tidal volume is 45 ml. Which of the following is the MOST appropriate ventilatory management for this patient? Decrease peak airway pressure to 25 cmH2O Decrease FiO2 to 50% Increase FiO2 to 90% Increase PEEP to 7 cmH2O

Decrease peak airway pressure to 25 cmH2O Due to the concern of aggressive ventilation on both the short-term survival as well as long-term outcomes of congenital diaphragmatic hernia, ventilatory strategies that employ small tidal volumes with permissive hypercapnia have gained widespread acceptance. Boloker, et al. suggested preservation of spontaneous ventilation, acceptance of a pre-ductal oxygen saturation of 90-95% with >80% tolerated if the infant appears comfortable, permissive hypercapnia of 60-65 mmHg, and peak inspiratory pressures < 25 cm H20.

A 65-year-old man with hypertension and diabetes is undergoing an open abdominal aortic aneurysmectomy in the operating room. The surgeon plans to use a suprarenal aortic cross clamp and requests fenoldopam. Which of the following hemodynamic changes is MOST characteristic of fenoldopam? Increased heart rate Decreased heart rate Increased arterial blood pressure Decreased arterial blood pressure

Decreased arterial blood pressure Fenoldopam is a selective dopamine-1 receptor agonist that has the benefits of improving renal blood flow, contributing to diuresis and natriuresis but has little or no alpha or beta receptor or dopamine 2 receptor agonist activity. Fenoldopam has hypotensive effects through reduction in peripheral vascular resistance, but does not exert much effect on the heart rate. It is commonly used in cardiac and aortic vascular surgery due to its antihypertensive and renal-protection effects.

A 4-week-old infant presents with tachycardia and tachypnea and is refusing to take anything by mouth. Chest x-ray reveals congenital emphysema on the left. What would be the most likely findings on physical exam on the affected side? Decreased breath sounds, hyper-inflation, hyper-resonance Decreased breath sounds, hyper-inflation, hypo-resonance Increased breath sounds, hyper-inflation, hyper-resonance Increased breath sounds, hyper-inflation, hypo-resonance

Decreased breath sounds, hyper-inflation, hyper-resonance Congenital lobar emphysema is a developmental anomaly of the lung that is characterized by hyperinflation of one or more of the pulmonary lobes. The left lung is more commonly involved and specifically the left upper lobe is the most commonly affected. Infants typically have tachypnea and increased work of breathing, and may have cyanosis. Recurrent pneumonia or poor feeding with failure to thrive are less frequent presentations that may occur in milder forms. Physical examination reveals decreased breath sounds and hyperresonance to percussion. A chest x-ray will reveal hyper-inflation.

A 56-year-old man with a history of tobacco abuse is scheduled to undergo thoracoscopic resection of a right upper lobe mass. Which of the following benefits of smoking cessation will be MOST immediate? Decreased airway secretions Decreased airway reactivity Decreased carboxyhemoglobin concentration Decreased incidence of postoperative pneumonia

Decreased carboxyhemoglobin concentration Smoking cessation results in many advantages to this patient; however, a reduction in significant morbidity and mortality does not occur for at least eight weeks. Indeed, surgery during the first four weeks of tobacco abstinence has been associated with increased pulmonary complications. While acute cessation is associated with decreased carboxyhemoglobin levels (and increased tissue oxygenation), airway reactivity and secretions increase.

Which of the following BEST explains why higher settings of desflurane are needed at higher altitudes to produce the same anesthetic effect? Decreased concentration of desflurane at higher altitudes Decreased partial pressure of desflurane at higher altitudes Decreased oxygen concentration at higher altitudes This statement is incorrect; equivalent settings produce the same anesthetic effect

Decreased partial pressure of desflurane at higher altitudes The partial pressure of most anesthetic vapors is dependent on temperature, not barometric pressure, and in an extremely cold environment more anesthetic vapor might be required. In one study, the delivered concentration of halothane increased with altitude, but its alveolar partial pressure remained constant. Although the concentration of the inspired volatile anesthetic was increased, the anesthetic effect remained unchanged at the given vaporizer setting. In other words, halogenated hydrocarbon vapors are delivered at a constant potency at constant temperature irrespective of altitude. Desflurane is the only exception to this rule. Unlike variable-bypass vaporizers, the Datex-Ohmeda (Steeton, UK) Tec 6 and Tec 6 plus vaporizers require manual adjustments of the concentration control dial at altitudes other than sea level to maintain a constant partial pressure of anesthetic.

Which of the following is the BEST option for treating hypotension related to arrhythmia during venous cannula placement prior to institution of cardiopulmonary bypass for the patient undergoing coronary artery bypass graft surgery? Cardioversion Discontinuation of mechanical stimulation Phenylephrine Volume replacement

Discontinuation of mechanical stimulation For cardiopulmonary bypass to be instituted, an arterial cannula is placed in the ascending aorta and venous cannulae are placed in the right atrium to drain deoxygenated blood from the patient. This blood is directed to the bypass circuit where it is oxygenated and CO2 is eliminated. Blood is then returned to the patient via the arterial cannula in the aorta. During placement of the venous cannula in the right atrium, surgical manipulation can and often does trigger arrhythmias: the most common being atrial fibrillation. Cessation or limited mechanical stimulation is recommended if arrhythmias occur. Often, this alone corrects the problem. Other maneuvers that may be successful include giving a fluid bolus, titrating vasoactive medications and cardioversion.

A 70-year-old man with a DDD-R pacemaker for a history of symptomatic bradycardiais undergoing an anterior cervical discectomy and fusion with somatosensory evoked potential (SSEP) and motor evoked potential (MEP) monitoring. The pacemaker should be reprogrammed to which of the following? Discontinue R function Dual chamber asynchronous pacing No reprogramming Ventricular asynchronous pacing

Discontinue R function "R" signifies rate responsiveness in the fourth position of the pacemaker designation code. Patients who are pacemaker-dependent are limited in their ability to exercise because of fixed rate (can't get their heart rates up). "R" function allows a pacemaker to speed up to satisfy increased metabolic demands (via motion, minute ventilation, temperature sensors) when the patient is exercising. However, for patients who are pacemaker-dependent, rate responsiveness function may be activated by perioperative events: fasciculations from succinylcholine, myoclonus from etomidate, vigorous surgical retraction, shivering, or SSEP/MEP neuromonitoring. This can cause undesired tachycardic pacing. So in cases like the one described, it is best to turn off the rate responsiveness function. Keeping the pacemaker in DDD is otherwise acceptable as long as a method of perfusion is assured, such as arterial blood pressure or pulse oximetry. One concern is often that the pacemaker will interpret artifact or electromagnetic inference from the bovie cautery as native heart rhythm and not initiate pacing when it is indicated, leaving the patient at risk for profound bradycardia or asystole. Placing the bovie pad away from the pacemaker generator and using bipolar cautery if needed are options to deal with that type of interference. Reprogramming to asynchronous mode risks R on T phenomenon (when the pacemaker cannot discern between artifact and native heart rhythm).

A 62-year-old otherwise healthy woman loses 1000 mL of blood rapidly during a partial hepatectomy. After adequate volume resuscitation, including the initiation of packed red blood cell transfusion, she becomes increasingly hypotensive and tachycardic. Her vital signs are BP 64/42 mmHg, HR 136 bpm, SpO2 98%, temperature 38.4°C. What is the MOST appropriate next step in management? Continue packed red blood cell transfusion Initiate fresh frozen plasma transfusion Continue volume resuscitation with tetrastarch Discontinue transfusion

Discontinue transfusion This patient has signs suggestive of an acute transfusion reaction. Despite adequate volume resuscitation and control of surgical bleeding, she is hypotensive, tachycardic and febrile after the initiation of packed red blood cell transfusion. The transfusion should be discontinued and the blood sent back to the blood bank for testing. Febrile transfusion reactions occur in 0.5% of RBC transfusions and 30% of platelet transfusions and are thought to be due to recipient antibodies directed against HLA antigens on donor WBC or platelets. Cytokines released from WBC in stored blood product (especially platelets) may also be a contributing factor. Patients experiencing a febrile reaction have increase in temperature of > 1 C as well as headache and back pain (not obvious in patient under general anesthesia) in addition to signs similar to allergic reaction. The febrile reaction is usually delayed up to 2 hours after the transfusion but is treated successfully with acetaminophen and diphenhydramine. Leukoreduction helps to reduce febrile transfusion reactions. An acute hemolytic transfusion reaction is usually the result of ABO blood group incompatibility and may be fatal. Patients having acute hemolytic reaction present with fever, dyspnea, chest pain, low back pain and sudden hypotension. Under general anesthesia only hypotension and fever may be apparent. Acute renal failure may result; the transfusion should be stopped and volume resuscitation with addition of mannitol or furosemide should be considered.

A 5-year-old with Duchenne's muscular dystrophy presents to preoperative clinic before elective bilateral lower extremity tendon lengthening. Which of the following is the MOST important for further preoperative assessment of this child? Complete Blood Count Electrolyte panel Electrocardiogram and echocardiogram History and Physical Exam

Duchenne's muscular dystrophy (DMD) is the most common and severe form of muscular dystrophy and is an X-linked recessive disease resulting in a mutation in the dystrophin gene. DMD has effects on many organ systems in addition to skeletal muscle, including the heart. Patients frequently develop a dilated cardiomyopathy from fatty infiltration of the myocardium. This may present initially on ECG as prominent Q waves, inverted T waves, or other changes. Echocardiography will show LV wall motion abnormalities as fibrosis progresses and indicates the disease is advancing. Some form of cardiac involvement is present in up to 90% of patients; therefore, a cardiac workup is most appropriate for this patient preoperatively.

A 50-year-old woman presents with chest pain, shortness of breath, and diaphoresis. She is found to have a large pericardial effusion. She has pulsus paradoxus. Which of the following accounts for the finding of pulsus paradoxus? Leftward shift of interventricular septum Rightward shift of interventricular septum Compression of left atrium Compression of right atrium

During spontaneous inspiration there is an exaggerated drop in the systemic blood pressure. Leftward shift of the interventricular septum accounts for the clinical sign of pulsus paradoxus.

During which of the following phases does MOST left ventricular filling occur? Atrial systole Early diastole Isovolumic relaxation Ventricular systole

Early diastole The left ventricle fills during diastole, when the left atrial pressure exceeds left ventricular pressure and the mitral valve opens. More than 60% of all left ventricular filling occurs during early diastole. Atrial systole is responsible for only about 15% of ventricular filling. The mitral valve is closed during isovolumic relaxation and ventricular systole and thus no ventricular filling occurs.

Which of the following monitoring modality is MOST important in the setting of tricyclic antidepressant (TCA) poisoning? Pulse oximetry Electrocardiogram Blood pressure PaCO2

Electrocardiogram TCA overdose can lead to lethargy, delirium, coma, and seizures. Tachycardia and hypotension may also develop. Attempt gastric lavage if history suggests recent large ingestion (10-20 mg/kg) and use activated charcoal. The ECG tracing is crucial in the management of TCA overdose. Give sodium bicarbonate if QRS duration > 100 ms (1-2 meq/kg bolus) and convert from bolus to infusion when the QRS complex narrows (targeting a pH of 7.5 to 7.55). If the pH becomes too alkaline but QRS complex widens then given 3% hypertonic saline to antagonize sodium channel blockade associated with TCAs. Use norepinephrine if vasopressors are necessary. Seizures need to be rapidly controlled with GABA agonists (benzos, propofol) b/c associated metabolic acidosis will rapidly worsen toxicity; don't give phenytoin which is a type IA anti-arrhythmic.

A 55-year-old woman with seasonal allergies and history of Hepatitis C presents for elective inguinal hernia repair. She has no known drug allergies. General anesthesia is induced with fentanyl, propofol, and lidocaine. Endotracheal intubation is facilitated with succinylcholine after a 5 mg defasciculating dose of rocuronium. After intubation, ventilation is difficult with high pressures with dense wheezes bilaterally. The non-invasive blood pressure cuff reads 55/30 mmHg. The patient's skin appears flushed. Which of the following medications should MOST likely be administered first? Albuterol Diphenhydramine Epinephrine Phenylephrine

Epinephrine This patient is most likely suffering anaphylaxis and epinephrine is the medication of utmost importance during anaphylaxis. It provides vasoconstriction of the vasculature to treat hypotension and bronchodilation to mitigate bronchoconstriction. In the operating room, epinephrine should be titrated in as soon as anaphylaxis is suspected. In addition to giving epinephrine, the presumed inciting agent should be discontinued or removed. The ACLS algorithm should be followed as necessary. Increase FiO2 to 100%. Consider manual ventilation. Intravenous fluids should be administered generously. Other adjuncts such as beta-agonists, corticosteroids, and histamine blockers can then be given.

A 55-year-old woman was recently diagnosed with restrictive lung disease. Which of the following findings would LEAST suggests the diagnosis of restrictive lung disease? Decreased vital capacity Decreased lung compliance FEV1/FVC < 0.7 Increased change in intrapleural pressure required to create same tidal volume

FEV1/FVC < 0.7 Restrictive lung disease is associated with a proportionate decrease in all lung volumes. Thus, vital capacity and functional residual capacity are both decreased. The FEV1/FVC ratio should be approximately normal due to an increased FEV1. The patient will have decreased lung compliance. This, in combination with the decrease in functional residual capacity, may lead to arterial hypoxemia. Patients may be prone to desaturate during apneic periods compared to normal patients. Larger changes in intrapleural pressure are required to create the same tidal volume. Thus, you may recognize a patient with restrictive lung disease taking more frequent breaths due to smaller tidal volumes.

Which of the following coagulation factors has the SHORTEST half-life? Factor II Factor VII Factor IX Factor X

Factor VII Factor VII has the shortest half-life: 4-6 hours. Factor IX has a half-life of about 24 hours. Factor X has a half-life of 25-60 hours and Factor II has a half-life of 50-80 hours.

Which of the following is the LARGEST branch of the lumbar plexus? Femoral nerve Lateral femoral cutaneous Sciatic nerve Obturator nerve

Femoral nerve The femoral nerve is the largest branch of the lumbar plexus and arises from the posterior division of the ventral rami of L2-L4.

Which of the following pharmacologic agents is MOST appropriate for the prevention of venous thromboembolism in the setting of suspected heparin-induced thrombocytopenia (HIT)? Subcutaneous heparin Low Molecular Weight Heparin Warfarin Fondaparinux

Fondaparinux Heparin-induced thrombocytopenia (HIT) occurs when IgG antibodies are formed to the heparin-platelet factor 4 (PF4) complex. These antibodies typically take 5 days to form. Interestingly, many people form heparin-PF4 antibodies, but only rarely do they lead to thrombocytopenia or thrombosis. The 4T scoring system has been proposed to determine the risk of HIT. It is based on four categories - Thrombocytopenia extent, Timing, Thrombosis symptoms, and alTernative explanations. If HIT is unlikely but suspected, it is reasonable to prove venous thromboprophylaxis with fondaparinux. Fondaparinux is related to heparin (it is synthetic pentasaccharide Factor Xa inhibitor) but does NOT have an O-methyl group that both UFH and LMWH have. Note, however, that this is an off label use of this drug. LMWH does appear to have a lower risk of HIT than UFH (RR 0.24) but this is not low enough to justify its use when HIT is expected. If HIT is strongly suspected and/or thrombosis is present, anticoagulation should be initiated using a direct thrombin inhibitor. Coumadin must be avoided in the setting of HIT as it may lead to "warfarin necrosis." Do not transfuse platelets as they may exacerbate the thrombosis.

Forms of mechanical ventilation that produce tidal volumes at or below anatomic dead space include: (Select 2) high-frequency oscillation inverse I:E ratio ventilation airway pressure release ventilation differential lung ventilation high-frequency positive-pressure ventilation pressure support ventilation

Forms of mechanical ventilation that produce tidal volumes at or below anatomic dead space include: high-frequency oscillation, high-frequency positive-pressure ventilation High-frequency oscillation (HFO) creates a to-and-fro gas movement in the airway at rates of 180 - 3000 times/min. High frequency positive-pressure ventilation is delivered at a rate of 60 - 120 breaths/min. Tidal volume is at or below anatomic dead space. High-frequency ventilation techniques may be useful in cases of bronchopleural and tracheoesophageal fistulas

Which of the following nerves provides sensory innervation to the pharynx? Internal branch of superior laryngeal nerve Recurrent laryngeal nerve External branch of superior laryngeal nerve Glossopharyngeal nerve

Glossopharyngeal nerve The glossopharyngeal nerve provides sensory innervation to the pharynx. The internal branch of the superior laryngeal nerve provides sensory innervation to the larynx above the level of the vocal cords. The recurrent laryngeal nerve provides sensory innervation to the larynx below the level of the vocal cords. The superior laryngeal and recurrent laryngeal nerves are both branches of the vagus nerve.

Which of the following is the narrowest portion of the pediatric airway? Cricoid ring Glottic opening Nasal opening Posterior pharynx

Glottic opening This is a challenging question that few people answer correctly! Most anesthesiologists think that the narrowest part of the pediatric airways is the cricoid ring, which is no longer thought to be true. Classically, the cricoid ring has been described as the narrowest part of the pediatric airway. Recent studies by Litman and Dalal measured airway dimensions in anesthetized children. They have shown that the glottic opening is the narrowest part of the pediatric airway, like the adult. Nevertheless, the cricoid ring is fixed, non-distensible, and prone to edema formation with a large endotracheal tube. A small change in airway diameter from edema contributes significantly to airway resistance in a pediatric (small) airway.

A 35-year-old woman who underwent orthotopic heart transplantation 2 years ago for nonischemic cardiomyopathy presents after a motor vehicle accident for exploratory laparotomy under general anesthesia. Intraoperatively, her blood pressure is 75/35 mmHg and heart rate is 90 bpm. After the administration of phenylephrine, which of the following hemodynamic responses do you MOST expect? HR decreased, BP increased HR decreased, BP no change HR no change, BP increased HR no change, BP no change

HR no change, BP increased After heart transplantation, the heart is completely denervated. The normal resting heart rate is relatively tachycardic at 90-100 bpm due to lack of vagal tone. Vagal bradycardic responses (to laryngoscopy, hypertension, carotid sinus massage) will also be absent. Over time, however, many patients require permanent pacemaker placement for treatment of significant bradycardia. After heart transplant, patients are not able to respond to demands for increased cardiac output with increased heart rate. Thus in this situation of a trauma with potentially significant blood loss, a normal patient would have tachycardia but a heart transplant patient has no change in heart rate, only hypotension. Instead for heart transplant patients, cardiac output is augmented by increased stroke volume. For this reason it is important to maintain adequate intravascular volume. The transplanted heart is not able to respond to medications that block the parasympathetic system. Bradycardia and hypotension have to be treated with medications that have a direct effect such as epinephrine and isoproterenol. Phenylephrine will result in increased blood pressure, but no change in heart rate. Indirect and mixed indirect/direct-acting drugs have minimal effect or have the effect of their direct components.

After an accidental needle stick with a contaminated needle, which of the following viruses carries the HIGHEST risk of transmission? HIV Hepatitis A Hepatitis B Hepatitis C

Hepatitis B Hepatitis B carries the greatest risk of transmission, with 37 to 62% of exposed workers eventually showing seroconversion and 22 to 31% showing clinical hepatitis B infection. The hepatitis C transmission rate has been reported at 1.8%, but newer, larger surveys have shown only a 0.5% transmission rate. The overall risk of HIV infection after percutaneous exposure to HIV-infected material in the health care setting is 0.3%. Hepatitis B prophylaxis Current CDC guidelines call for the administration of hepatitis B immune globulin (HBIG) and/or hepatitis B vaccine. While the efficacy of the combination has not been evaluated in the needlestick injury setting, it has been shown to be the most efficacious approach in the perinatal setting. The approach has no contraindications during pregnancy and lactation. Hepatitis C prophylaxis CDC guidelines acknowledge that there is no active post-exposure prophylaxis for HCV. There is some evidence that treatment with interferon alfa-2b may be beneficial preventing chronic hepatitis. HIV prophylaxis CDC guidelines generally recommend a post-exposure prophylaxis protocol with 3 or more antiviral drugs, when it is known that the donor was HIV positive; however, when the viral load was low and none of the above noted risk factors are met, the CDC protocol utilizes 2 antiviral drugs.

A 21-year-old healthy man is mechanically ventilated after an ORIF of a tibial plateau fracture. The addition of 15 cmH2O PEEP would MOST likely have which of the following cardiovascular effects? Increased LV afterload Increased LV preload Increased RV afterload Increased RV preload

High levels of positive end-expiratory pressure (PEEP) result in predictable changes in cardiac output and blood pressure. The primary impact of PEEP is a progressive decrease in venous return and ultimately a decrease in RV preload. While PEEP decreases RV preload by impairing systemic venous return, it will also increase RV afterload, primarily by impeding RV ejection. This ultimately results in a decrease in RV cardiac output, which results in a decrease in LV preload. LV afterload can actually be reduced by PEEP.

A 62-year-old woman with hyperparathyroidism underwent a parathyroidectomy 24 hours ago. She complains of numbness and tingling in her hands and feet and mild shortness of breath. Which of the following laboratory abnormalities is MOST consistent with her symptoms? Hypocalcemia Hypokalemia Hypermagnesemia Hyperphosphatemia

Hypocalcemia Hypocalcemia is a common complication associated with parathyroidectomy for hyperparathyroidism. In hyperparathyroidism, PTH increases bone formation and resorption with a net efflux of calcium from bone (remember PTH "trashes" the bones). Sudden removal of PTH causes an imbalance between osteoclast mediated bone resorption and osteoblast mediated bone formation. This leads to an increase in bone uptake of calcium, phosphate, and magnesium. This "hungry bone syndrome" upsets the equilibrium between calcium efflux from bone and influx during bone remodeling. Thus the laboratory abnormalities seen after parathyroidectomy with hungry bone syndrome include hypocalcemia, hypophosphatemia, hypomagnesemia, and hyperkalemia. Hyperkalemia is thought to reflect increased bone efflux but is not known for certain and most commonly affects dialysis-dependent patients who undergo a parathyroidectomy. Signs and symptoms of hypocalcemia after parathyroidectomy include neuromuscular irritability in the form of tetany (Chvostek's sign: tapping inferior part of zygoma causes facial spasms or Trousseau's sign: carpal spasm by inflating BP cuff, as well as bronchospasm or laryngospasm), ECG changes, and seizures. "CATS go numb" is a common mnemonic to help remember the signs and symptoms of hypocalcemia: Convulsions, Arrhythmias, Tetany and numbness/paresthesias in hands, feet, mouth and lips. ECG changes include QT prolongation that predisposes to cardiac electrical instability and increased risk for torsades de pointes.

During a craniotomy, a patient develops intraoperative diabetes insipidus. Which of the following is the MOST likely cause of central diabetes insipidus and what is the MOST appropriate treatment? Medullary compression; vasopressin Pontine injury; vasopressin Hypertonic saline; diuresis Hypophyseal injury; vasopressin

Hypophyseal injury; vasopressin Central diabetes insipidus is due to injury to the neurohypophysis and subsequent reduction in vasopressin secretion. Clinically, patients have high volume dilute urine with a rising serum sodium. Patients with chronic untreated diabetes insipidus must drink large volumes of liquid to compensate. Treatment is generally with nasal ddAVP. Acute diabetes insipidus in the operating room or ICU is usually initially treated with intravenous vasopressin or ddAVP.

Which of the following anomalies is MOST likely to be associated with congenital diaphragmatic hernia? Horseshoe kidney Hypoplastic left heart Radial limb anomalies Vertebral anomalies

Hypoplastic left heart A congenital diaphragmatic hernia (CDH) is an early developmental defect that results in the extrusion of intraabdominal organs (i.e. stomach, small intestines, kidney, liver) into the thoracic cavity. Significant cardiac disease is associated with at least 10% of of patients with CDH. Survival for patients with cardiac disease is significantly lower than for patients with normal cardiac anatomy. Patients with CDH and univentricular cardiac anatomy have a poor prognosis.

A 68-year-old man undergoes right colectomy for colorectal cancer. He had been taking clopidogrel, which was held for one week prior to surgery. Intraoperatively he is transfused one unit of PRBCs and one unit of platelets. On post-operative day 3, his hemoglobin drops from 10 to 8.2 mg/dL. He is hemodynamically stable and only complains of mild back pain. He is transfused 1 unit of PRBCs and follow-up Hgb is 8.4 mg/dL. What is the MOST likely cause for his anemia? Dilution of blood by maintenance IV fluids Inadequate surgical hemostasis exacerbated by preoperative clopidogrel Carcinoma-induced coagulopathy Immune-mediated reaction

Immune-mediated reaction This patient most likely has a delayed antibody-mediated hemolytic transfusion reaction from the PRBCs given during his surgery. Hemolysis is the most likely cause for the patient's anemia at this time due to lack of signs suggestive of acute blood loss. Such reactions can occur 3-21 days post-transfusion. Laboratory values such as elevated unconjugated bilirubin help lend evidence to this mechanism for his anemia.

*FLIP FOR QUESTION Answer = A - Curve A best represents the effects of changing oxygen tensions on cerebral blood flow. Hypoxemia causes a significant increase in CBF to meet the brain's metabolic demand. Hyperoxia, however, causes little change in CBF. - Barash pg 999

In the graph of cerebral blood flow below, PaO2 would best be represented by curve: A, B, C, or D

An 80-year-old man has been in the ICU for 12 hours following an aortic valve replacement and has been on nitroprusside for control of hypertension. Which of the following is MOST responsible for the toxicity caused by nitroprusside? Accumulation of thiosulfate and cyanide Formation of cyanmethemoglobin Inactivation of cytochrome oxidase Production of thiocyanate

Inactivation of cytochrome oxidase After nitroprusside enters red blood cells, electron transfer occurs and the compound dissolves into 5 cyanide ions and a nitroso group. The cyanide ions can undergo 3 different reactions: forming cyanmethemoglobin; binding with thiosulfate; or interacting directly with cytochrome oxidase. The inactivation of cytochrome oxidase is responsible for the uncoupling of mitochondrial oxidative phosphorylation and shift from aerobic to anaerobic metabolism. This causes production of lactic acid and the symptoms of cyanide toxicity including metabolic acidosis, arrhythmias, tachycardia, hypertension, neurologic dysfunction (confusion) and increased mixed venous oxygen content. One other key sign of cyanide toxicity is tachyphylaxis or the resistance to the effects of nitroprusside to reduce blood pressure.

A 40-year-old man was diagnosed with hypertension and his primary care physician started him on hydrochlorothiazide for blood pressure control. He presents for elective laparoscopic cholecystectomy. Which of the following electrolyte disturbances is MOST common in patients taking hydrochlorothiazide? Increased calcium Increased magnesium Increased potassium Increased sodium

Increased calcium Hydrochlorothiazide (HCTZ) is a sulfonamide derivative in the thiazide class of diuretics. It is often used as a first line treatment for hypertension. It blocks the sodium chloride transporter in the distal convoluted tubule of the kidney. Thus, HCTZ causes a natriuresis along with chloride loss. Other effects include reductions in potassium and magnesium. HCTZ increases calcium due to reabsorption of calcium also at the distal convoluted tubule. Glucose levels can rise if potassium is supplemented with use of HCTZ. In addition, LDL cholesterol, total cholesterol and triglycerides also increase.

Which of the following physiologic changes that improve oxygen delivery to the fetus during normal pregnancy is LEAST likely? Increased cardiac output Increased hematocrit Increased PaO2 Maternal hyperventilation

Increased hematocrit Several physiologic alterations normally occur to create favorable conditions for oxygen delivery to the fetus. Minute ventilation and PaO2 increase via increased tidal volume and respiratory rate, to compensate for the increase in oxygen consumption and carbon dioxide production. Oxygenated hemoglobin is delivered swiftly in larger amounts as a result of increased cardiac output. Although the number of intravascular red cells is increased 30%, a concurrent 45% increase in blood volume at term results in relative hemodilution. Thus, although higher total amounts of red cells and hemoglobin are produced for oxygen delivery, there is actually a decreased hematocrit ("physiologic anemia of pregnancy").

Regarding neuromuscular blockade in the ICU, which of the following statements is MOST true? Decreased incidence of polyneuropathy and myopathy, potentially harmful in severe ARDS Decreased incidence of polyneuropathy and myopathy, potentially useful in severe ARDS Increased incidence of polyneuropathy and myopathy, potentially useful in severe ARDS Increased incidence of polyneuropathy and myopathy, potentially harmful in severe ARDS

Increased incidence of polyneuropathy and myopathy, potentially useful in severe ARDS There has been a trend towards decreased paralytic use in the ICU as it has been increasingly recognized that they may be associated with critical illness polyneuropathy and myopathy (especially when co-administered with steroids) and also as the deleterious effects of prolonged sedation and intubation are better appreciated. However, there may occasionally be compelling reasons to administer paralytics in the setting of ARDS, where they may lower the consumption of oxygen and allow for ultra low tidal volumes. The ARDS et Curarisation Systematique (ACURASYS) study, published in NEJM in 2010, allowed TV as low as 4 cc/kg and revealed a lower mortality in patients with severe ARDS who received 48 hours of paralysis with cisatracurium, and found no increase in myopathy.

Compared to "plain" local anesthetic without epinephrine, a pre-mixed local anesthetic with epinephrine solution is MOST associated with which of the following? Little effect on peak plasma levels Decreased sensory block Increased onset time Decreased cardiac output

Increased onset time Commercially prepared solutions with epinephrine have a lower pH than those in which it is freshly added. A lower pH results in a higher percentage of ionized drug molecules. It is the non-ionized form that easily crosses the lipid membrane; therefore the onset will be delayed.

A patient with preeclampsia is receiving magnesium and has a magnesium level of 10 mEq/L. A magnesium level of 10 mEq/L is MOST associated with which of the following? Heart block Increased sensitivity to both depolarizing and non-polarizing muscle relaxants Therapeutic range for seizure prophylaxis No side effects

Increased sensitivity to both depolarizing and non-polarizing muscle relaxants Magnesium, administered intravenously, is given for seizure prophylaxis to parturients who present with preeclampsia. While normal serum levels of magnesium are 1.2-2.0 mEq/L and therapeutic range is 4-8 mEq/L, EKG changes, including prolonged PR intervals and widened QRS complexes, can develop at serum levels of 5-10 mEq/L. Magnesium depresses activity at the neuromuscular junction, and therefore loss of deep tendon reflexes can occur as a sign of magnesium toxicity. Therefore, patients with elevated serum magnesium levels can be more sensitive to both depolarizing and non-depolarizing muscle relaxants.

Which of the following is MOST likely to INCREASE a patient's mixed venous oxygen saturation? Administering a beta-blocker Inducing general anesthesia with a muscle relaxant Reducing the FiO2 so that the patient's SpO2 decreases 100% to 95% Removing 2 units of blood during acute normovolemic hemodilution (ANH)

Inducing general anesthesia with a muscle relaxant SvO2 = SaO2 -- [(VO2)/(Hbg x 1.36 x Q)], where SvO2 is mixed venous oxygen saturation, SaO2 is arterial oxygen saturation, VO2 is oxygen consumption, Hgb is hemoglobin, and Q is cardiac output. General anesthesia with a muscle relaxant would be expected to decrease VO2, thereby increasing SvO2. (Granted, this assumes the patient does not arrest during induction.) Administering beta blockers would be expected to decrease cardiac output and therefore lower SvO2. Lowering a patient's hemoglobin with ANH would be expected to decrease SvO2, as would decreasing their SpO2.

A 4-year-old boy with acute epiglottitis is receiving humidified oxygen by facemask. He has a functioning IV in his right hand. Which of the following is the MOST appropriate induction technique? Inhalation induction with sevoflurane Intravenous sedation with propofol Intravenous induction with propofol and rocuronium Intravenous induction with ketamine and rocuronium

Inhalation induction with sevoflurane Acute bacterial epiglottitis usually presents with airway compromise and may progress to complete airway obstruction with airway instrumentation. The most conservative management is to anesthetize the patient and perform one direct laryngoscopy after the patient and the airway is satisfactorily anesthetized. This should be performed in an operating room where personnel and equipment are immediately available for an emergency tracheostomy. A smaller (reduce by 0.5) endotracheal tube should be available. Sedation is relatively contraindicated because this may compromise the patient's airway further. Maintaining spontaneous ventilation is a hallmark of the management of the difficult pediatric airway. Providing muscle relaxation may result in a "can't intubate, can't ventilate" scenario and is not recommended. Ketamine may also increase the secretions that are already present and decrease visualization of the glottis opening further.

A 50-year-old woman is undergoing a mitral valve repair. She has been taking garlic prior to surgery as an herbal supplement to reduce blood pressure and improve her overall health. She has significant mediastinal blood loss after separation from bypass. Which of the following effects can MOST be attributed to garlic? Increased fibrinolysis Increased consumption of coagulation factors Inhibition of the prothrombinase complex Inhibition of platelet aggregation

Inhibition of platelet aggregation The use of herbals has increased but herbal medicines are not regulated by the FDA. This makes it difficult to know what exact substances patients are ingesting and there are a multitude of side effects that can result from herbals. Garlic is used by some to treat hypertension and to reduce lipids, but the organosulfur compound irreversibly inhibits platelet aggregation. The effects of garlic require 7 days of discontinuation prior to surgery to reduce postoperative bleeding. Ginkgo causes milder bleeding than garlic and its effects are attributed to inhibition of platelet activating factor. Ginkgo should be stopped 36 hours prior to surgery. Ginseng can inhibit platelet aggregation as well and should be stopped 7 days prior to surgery.

A 75-year-old woman presents for combined mitral valve repair and aortic valve replacement after CABG 10 years ago. She is deemed to be high risk for bleeding after cardiopulmonary bypass and during the postoperative period in the ICU. You plan to administer tranexamic acid prior to commencement of cardiopulmonary bypass. Which of the following BEST describes the mechanism of tranexamic acid? Activates platelets Improves fibrin concentration Increases thrombin Inhibits activation of plasmin

Inhibits activation of plasmin Prohemostatic interventions for patients having cardiac surgery at high risk of bleeding are very important and attack the coagulation cascade at many different points. Tranexamic acid, epsilon aminocaproic acid and aprotinin (no longer used in the US due to safety concerns) work by inhibiting conversion of plasminogen to plasmin and plasmin release. Thus they have an anti-fibrinolytic effect and attempt to preserve clot formation (reduce inappropriate breakdown of thrombus). Other treatments used include DDAVP which increases the release of von Willebrand factor from the endothelium in order to increase platelet activation; prothrombin complex concentrates and recombinant factor VIIa which increase thrombin formation (and thus improve clot formation); and fibrinogen concentrates or cryoprecipitate which increases fibrinogen and thus the fibrin component of clot formation.

Systemic levels of mepivacaine would MOST likely be the greatest 10 minutes after which of the following regional techniques using an equal volume of 1.5% mepivacaine? Brachial plexus Caudal Epidural Intercostal

Intercostal Systemic absorption of a local anesthetic is determined by the site of injection, the concentration and volume of local anesthetic, the addition of vasoconstricting additives, and the pharmacologic profile of local anesthetic. Generally speaking, the more vascular the region the higher the blood levels after injection. Absorption from the intercostal space is rapid, with plasma concentrations peaking as quickly. Many people use the acronym "ICE-BS" (Intercostal -- Caudal -- Epidural - Brachial plexus - Spinal) to remember fastest to slowest systemic absorption.

A 40-year-old woman with multiple sclerosis presents for a vaginal hysterectomy. Which of the following has been MOST associated with perioperative multiple sclerosis exacerbation? Intraoperative hyperthermia Ketorolac for perioperative analgesia Neuromuscular blockade with rocuronium Reversal of neuromuscular blockade with neostigmine

Intraoperative hyperthermia Multiple sclerosis (MS) is a central demyelinating neurologic disease. MS is characterized by periods of relapse and remission and variable presentation. It is said to "vary in space [effected areas] and time [relapse and remission]." Patients may be either sensitive or insensitive to neuromuscular blockade and succinylcholine may result in hyperkalemia; however, there is no evidence that neuromuscular blockade or its reversal is associated with perioperative exacerbation. Ketorolac has not been implicated in perioperative exacerbations. Hyperthermia has been associated with perioperative exacerbations and increases as little as 1C may result in disease exacerbation.

Which of the following drugs is included in Step 1 in the World Health Organization (WHO) analgesic ladder? Codeine Ketoprofen Propoxyphene Tramadol

Ketoprofen The World Health Organization (WHO) analgesic ladder was established in 1986 to guide physicians developing treatment plans for cancer pain. In general terms, Step 1 includes Non-opioid analgesics, with or without adjuvants. Step 2 includes "weak" opioids (such as codeine, propoxyphene, and tramadol), with or without adjuvants. Step 3 includes "strong" opioids (such as morphine, fentanyl, and methadone) and non-opioids, without or without adjuvants. Ketoprofen is the only non-opioid listed.

Which of the following is MOST correct regarding the treatment of known venous thromboembolism with either low molecular weight heparin (LMWH) or unfractionated heparin (UFH)? LMWH more efficacious but with more side effects LMWH more efficacious and with fewer side effects UFH more efficacious but with more side effects UFH more efficacious and with fewer side effects

LMWH more efficacious and with fewer side effects While it has been firmly established the LMWH is superior to UFH for the prevention of VTI, it appears that if venous thromboembolism does develop, fixed dose LMWH is more effective and safer than UFH (based on 23 studies including 9587 patients) for the treatment of VTE. These data suggest statistically significant reductions in thrombotic complications, more frequent reduction in thrombus size, fewer major hemorrhages, and lower mortality.

Laminar flow in the airway occurs in the: (Select 2) trachea main stem bronchi terminal bronchiole 3rd generation bronchus respiratory bronchiole

Laminar flow in the airway occurs mostly in the: terminal bronchiole, respiratory bronchiole Flow in the larger airways is mostly turbulent. Laminar flow normally occurs only distal to small bronchioles (< 1mm). The Reynolds number is used to predict the type of airway flow; a low Reynolds number (< 1000) is associated with laminar flow, whereas a high value (> 1500) is associated with turbulent flow.

A 50-year-old woman presents with chest pain, shortness of breath and diaphoresis. She has pulsus paradoxus and is hypotensive. Which of the following echocardiographic signs is MOST SPECIFIC for cardiac tamponade? Left atrial collapse Right atrial collapse Right ventricular wall inversion Left ventricular wall inversion

Left atrial collapse Echocardiographic findings include size of effusion and the effects of the effusion on cardiac filling. Small effusion measures < 9mm, moderate effusion measures 10-19 mm, and large effusion measures > 20 mm. Most characteristic signs are chamber collapse, mostly the RA and RV. During diastole the RV free wall invaginates and at end-diastole the RA wall invaginates. RA collapse is common, but is more sensitive when it lasts for at least 30% of cardiac cycle. LA collapse occurs in 25% of pts with tamponade and is very specific. LV collapse is rare due to muscular nature of LV wall, but bulging of IVS from right to left ventricle is specific for tamponade (and accounts for pulsus paradoxus).

Which of the following symptoms would be MOST expected in a 64-year-old man with an acute right anterior cerebral artery (ACA) stroke? Left leg weakness Left hand weakness Left CN VII palsy Hoarseness

Left leg weakness The left and right anterior cerebral arteries (ACA) supply blood to most medial portions of the frontal lobes and superior medial parietal lobes. This portion of the brain supplies innervation primarily to the lower extremities, most of the corpus callosum, the anterior portions of the basal ganglia and internal capsule, and the olfactory bulb and tract. Thus, patients with an acute ACA stroke classically present with hemiparesis or hemiplegia of the contralateral lower limbs and pelvic floor musculature. Patients may also develop an apraxia (secondary to involvement of the corpus callosum), anosmia (secondary to involvement of the olfactory bulb and tract), and urinary incontinence.

Which of the following ligaments is traversed when using a paramedian approach to the neuraxis? Interspinous ligament Ligamentum flavum Posterior longitudinal ligament Supraspinous ligament

Ligamentum flavum The supraspinous and interspinous ligaments are both midline structures and are not traversed in a paramedian approach. The only structure traversed in both midline and paramedian approaches is the ligamentum flavum. The posterior longitudinal ligament is anterior to the spinal cord and not traversed in either the paramedic or midline approaches.

Which of the following statements regarding the pharmacology of heparin is MOST correct? Warfarin increases the synthesis of endogenous heparin Low Molecular Weight Heparin preferentially inhibits Factor Xa Heparin is a highly sulfated protein molecule with negative charge Protamine reverses heparin action as a competitive antagonist on the receptor binding site

Low Molecular Weight Heparin preferentially inhibits Factor Xa Heparin is a carbohydrate containing glucuronic acid residues. It has negatively charged sulfate groups "two per uronic acid residue"making it one of the strongest acids found in living things. The major inhibitor of thrombin, factors IXa, and factor Xa is AT III. UFH accelerates the formation of the thrombin-AT complex by 2000-fold and accelerates formation of the factor Xa-AT complex by 1200-fold. In contrast, LMWH fragments preferentially inhibit factor Xa. Protamine is a positively charged polypeptide that combines with the negatively charged heparin to form stable complexes resulting in neutralization of heparin activity. Warfarin acts as an anticoagulant by inhibiting vitamin K epoxide reductase.

You are in the maintenance phase of a 6-hour anesthetic for craniotomy for the resection of a craniopharyngioma. Urine output abruptly increases to over 1 liter/hour. What is the most appropriate next step in this patient's management? Maintain euvolemia and watch the base deficit Maintain euvolemia and check serial serum sodium levels Reduce fluid administration to reduce urine output Target hypervolemia by keeping ahead of urine output

Maintain euvolemia and check serial serum sodium levels In this scenario, the anesthesiologist must be concerned about central diabetes insipidus. Euvolemia must be maintained and serum sodium followed (generally every 30-60 min) to look for rising sodium levels.

A 60-year-old man with a medically refractory seizure disorder has a generalized tonic clonic seizure complicated by aspiration pneumonia. He improves and his respiratory mechanics are compatible with extubation by ICU day #6. He is awake and alert; however, he has no leak around the endotracheal tube with the cuff deflated. What is the MOST appropriate management? Elective tracheostomy Proceed with extubation Methylprednisolone therapy prior to extubation Extubate over a tube changer, to facilitate re-intubation if necessary

Methylprednisolone therapy prior to extubation While many practitioners would opt for extubation in this scenario, there is high level evidence for the efficacy of methylprednisolone in reducing postextubation laryngeal edema and the incidence of re-intubation.

Which of the following MOST accurately describes respiratory changes in pregnancy? Arterial blood gas analysis of the parturient can be expected to show a pH between 7.45-7.50 At term, minute ventilation is increased by 70% over pre-pregnancy levels Minute ventilation increases by approximately 50% during the first trimester PaO2 at full term is increased by 40% over pre-pregnancy levels

Minute ventilation increases by approximately 50% during the first trimester This is a challenging question that few people answer correctly! Most anesthesiologists think that an arterial blood gas analysis of the parturient can be expected to show a pH between 7.45-7.50, but a normal arterial pH that is typically 7.42-7.44. Increased CO2 production and circulating progesterone levels are thought to be the cause of increased minute ventilation during pregnancy. Minute ventilation is increased by 50% above pre-pregnancy levels during the first trimester and remains constant for the remainder of the pregnancy. The increase in minute ventilation is mainly due to increased tidal volumes and slightly due to increased respiratory rate. A respiratory alkalosis does result, but is partly compensated for by increased renal excretion of bicarbonate, leading to an arterial pH that is typically 7.42-7.44.

Which of the following can be MOST expected from mask ventilation of the full-term parturient? Easier due to increased neck range of motion Easier due to increased chest wall compliance More difficult secondary to upper airway edema More difficult secondary to left uterine displacement positioning

More difficult secondary to upper airway edema Anesthesiologists need to account for the changes of the upper airway during pregnancy and plan accordingly. Capillary engorgement in the mucosal lining of the upper respiratory tract leads to tissue edema in a normal full-term pregnant patient resulting in smaller caliber upper airway. Increased tissue friability can be expected and extra care should be taken during any instrumentation of the mouth or upper airway, such as suctioning, placement of an oral airway or direct laryngoscopy. Active pushing leading to venous engorgement can exacerbate tissue edema along with the above-described changes. Weight gain during pregnancy can also make mask ventilation more challenging.

An 8-year-old, obese girl with severe obstructive sleep apnea is taken to the PACU after a tonsillectomy and adenoidectomy. On arrival, she is tachycardic, dyspneic, tachypneic, and anxious. Her SpO2 is 89% despite 4L nasal cannula. Auscultation reveals occasional wheezes bilaterally. A chest radiograph reveals diffuse interstitial and alveolar infiltrates. Given these findings, which of the following is the MOST likely diagnosis? Acute respiratory distress syndrome Negative pressure pulmonary edema Pulmonary aspiration Pulmonary hemorrhage

Negative pressure pulmonary edema Postoperative complications associated with tonsillectomy include pain, nausea and vomiting, dehydration, and post-tonsillectomy hemorrhage. Acute postoperative pulmonary edema, albeit infrequent, may occur when airway obstruction from tonsillar hypertrophy is relieved. Normal pleural pressures range from -2.5 to -10 cmH2O during inspiration. In a child with tonsillar hypertrophy and airway obstruction, pleural pressures may be as negative as -30 cmH2O. This may lead to pulmonary capillary wall dysfunction. Due to the negative pressure gradient during inspiration, a concomitant increased in venous return exists, but due to the microvasculature wall disruption, transudation of fluid into the alveolar space can occur. To counterbalance this potentially deleterious condition, patients with chronic airway obstruction develop an adaptive mechanism by which they generate positive intrapleural and alveolar pressures during exhalation, decreasing pulmonary venous return. Post-tonsillectomy, the obstruction has been relieved and the adaptive mechanism has been lost. This leads to decreased airway pressures during exhalation, increased venous return, increased pulmonary hydrostatic pressure, and then pulmonary edema.

Assuming no other clinical risk factors, which of the following conditions should MOST be evaluated and treated prior to a 65-year-old patient undergoing elective hip replacement? Moderate mitral regurgitation resulting from a myocardial infarction 6 months ago New asymptomatic atrial fibrillation with a heart rate of 120 NYHA class III heart failure symptoms that have been present for the last year An uncomplicated myocardial infarction 2 months ago

New asymptomatic atrial fibrillation with a heart rate of 120 New onset atrial fibrillation with an uncontrolled heart rate is an active cardiac condition (significant arrhythmia) that should delay elective surgery until further workup is performed and the ventricular rate better controlled. Mild or moderate valvular disease, old myocardial infarctions (>30 days out), and controlled heart failure should be appropriately managed, but do not require delay of surgery for further work-up.

A 24-year-old man with a history of T4 paraplegia is undergoing his first anesthetic for a urinary diversion due to a neurogenic bladder. As the surgeon makes the incision, his BP reaches 220/110 mmHg with a heart rate of 45 bpm. Which of the following is the MOST appropriate first step in treatment? Atropine Dantrolene Fentanyl Nitroprusside

Nitroprusside In a patient with a history of a spinal cord lesion higher than T7, marked hypertension and bradycardia is concerning for autonomic hyperreflexia. Normally, descending inhibitory impulses travel down the spinal cord to block reflex arcs to cutaneous, visceral, or proprioceptive stimuli. This arc is disrupted in spinal cord injury and can lead to autonomic instability, most notably severe hypertension followed by a sustained vagal response including bradycardia, vasodilation, and cutaneous flushing. Treatment is supportive, including stopping the inciting stimulus (ask surgeons to pause) and lowering the blood pressure to normal levels via vasodilators and assuring adequate levels of anesthesia.

A 65-year-old man with a history of coronary artery disease and previous CABG has a 6 cm abdominal aortic aneurysm (AAA). He presents to the OR for elective open repair of his AAA. Which of the following medications will be MOST likely to improve his myocardial function during the aortic cross-clamping? Nitroprusside Phenoxybenzamine Epinephrine Norepinephrine

Nitroprusside Principles of hemodynamic management during the period of aortic cross-clamping in patients with decreased myocardial reserve include reduction in afterload with arteriolar dilators such as nitroprusside and reduction in preload with venodilators such as nitroglycerin. Phenoxybenzamine is longer-acting, orally administered alpha-blocker which is often used in treatment of hypertension in patients with pheochromocytoma. It is not appropriate for use in AAA repair. Care should be taken with reduction in blood pressure in order prevent worsening of visceral ischemia distal to the aortic occlusion. Vasoconstrictors such as norepinephrine and epinephrine may be useful with removal of the aortic cross-clamp in low vascular resistance states. Epinephrine is not usually required as it often increases heart rate and myocardial oxygen consumption while increasing myocardial contractility. Other helpful management options after the cross clamp is removed include volume administration, treatment for hyperkalemia, acidosis and arrhythmias.

inhaled agents from highest vapor pressure to lowest

Nitrous Oxide, Des, Iso, Sevo

Regarding critically ill patients that require mechanical ventilation in the ICU, early tracheostomy after 4 days of mechanical ventilation compared to late tracheostomy after 10 days of mechanical ventilation is MOST associated with which of the following? Lower mortality Higher mortality No difference in mortality No difference in mortality but more ventilator days

No difference in mortality A previously utilized rule of thumb suggested that after 1 week of intubation, if extubation does not appear likely within a week, place the tracheostomy (because the clinical course at one week seems to be predictive of final outcome [1]). The TracMan trial randomized 909 patients to early tracheostomy (within 4 days) or late tracheostomy (after 10 days if still indicated) and found no difference in the primary outcome (30-day mortality) or other secondary outcomes [2].

You have designed a study that compares average postoperative opioid consumption after three different anesthetic techniques. After informed consent, 120 patients undergoing total knee arthroplasty will be randomly assigned to one of the three groups. Which of the following statistical tests can be BEST used to compare the means of three groups? Chi-square One-way ANOVA Paired t-test Unpaired t-test

One-way ANOVA One-Way Analysis of Variance (ANOVA) simultaneously compares the differences among population means of more than two independent groups for a one-factor experiment. Chi-square test for categorical variables determines whether there is a difference in the population proportions between two or more groups. The unpaired t-test compares the population means between two independent (and normally distributed) groups. The paired t-test examines repeated measurements obtained from the same set of individuals. The objective of the analysis is to show that any differences between two measurements of the same individuals are due to different treatment conditions. This approach is based on the theory that the same individuals will behave alike if they are treated alike and is frequently used in crossover studies.

A 60-year-old man is undergoing a Type A aortic dissection repair under deep hypothermic circulatory arrest. His current temperature is 24°C and his uncorrected ABG shows pH 7.40, PaCO2 40 mmHg and PaO2 250 mmHg. Which of the following changes would you MOST expect If you were to adjust his ABG to his core body temperature of 24°C? PaCO2 increased, PaO2 increased PaCO2 decreased, PaO2 decreased PaCO2 increased, PaO2 decreased PaCO2 decreased, PaO2 increased

PaCO2 decreased, PaO2 decreased As the temperature of blood decreases, the solubility of gases increases such that the partial pressure of CO2 and O2 decrease. Thus, a respiratory alkalosis develops as temperature decreases. The ABG appears normal at the temperature at which it is measured (37 °C) on the ABG machine; however, when corrected the patient's core temperature of 24°C he will have a respiratory alkalosis with PaCO2 and PaO2 that are decreased compared to the normal.

Which of the following is the MOST likely side effect of inhaled epoprostenol (Flolan)? Increased nitrogen oxides Methemoglobinemia Platelet dysfunction Systemic hypotension

Platelet dysfunction Like inhaled nitric oxide (NO), inhaled epoprostenol primarily affects the pulmonary vasculature, leading to improved RV function and better VQ matching with minimal systemic hypotension. Unlike NO, epoprostenol (Flolan) does not lead to the production of methemoglobin or nitrogen oxides, thus the delivery system can be much simpler (NO delivery systems are complex, because excess NO can be harmful). Interestingly, epoprostenol does appear to have some anti-platelet effects, mainly inhibition of platelet aggregation. Classically it has been taught that the clinical effects of this are mild; however, a recent analysis of epoprostenol use in patients undergoing LVAD implantation at the University of Virginia showed a significant increase in chest tube output associated with epoprostenol use.

Positive end expiratory pressure (PEEP): decreases dead space increases venous return to the heart decreases intrapulmonary shunting decreases extravascular lung water

Positive end expiratory pressure (PEEP): decreases intrapulmonary shunting The major effect of PEEP is to increase FRC and tidal ventilation above the closing capacity. This results in a decrease in intrapulmonary shunting. Neither PEEP nor CPAP decrease extravascular lung water. By increasing intrathoracic pressure, PEEP decreases venous return to the heart

A 75-year-old woman complains of loss of vision in one eye. She reports that she first noticed blurred vision in the right eye. This progressed to her bumping into objects on the right side. When she is asked to read, she can only read one half of sentences on a page. She has normal strength, coordination, reflexes and tone. Her cranial nerve exam shows that she cannot see a moving finger on the right field in either eye. An occlusion of which major vessel is associated with these findings? Basilar artery Internal carotid artery Middle cerebral artery Posterior cerebral artery

Posterior cerebral artery The description of this patient suggests that she has a right visual field deficit, in both eyes, as opposed to monocular blindness (the classic "amaurosis fugax" typically due to emboli from an internal carotid artery source). Although patients may describe not being able to see out of one eye, a cursory physical exam will reveal that the eye can see, but there may be a significant field cut in both eyes. The neurological term for this sign is homonymous hemianopsia, and it is most commonly due to occipital lobe injury from a posterior cerebral artery infarction. Although field cuts can also occur with temporal or parietal lesions, there tends to be other associated findings not seen in this patient (such as neglect).

Which muscle is the only abductor of the vocal cords? Cricothyroid muscle Thyroarytenoid muscle Posterior cricoarytenoid muscle Lateral cricoarytenoid muscle

Posterior cricoarytenoid muscle This is a challenging question that few people answer correctly! Most anesthesiologists do not understand the muscles anatomy of the larynx. The only abductor ("opener") of the vocal cords is the posterior cricoarytenoid muscle. All muscles of the larynx apart from the cricothyroid muscle are innervated by the recurrent laryngeal nerve; therefore, the only abductor muscle of the vocal cords is innervated by the recurrent laryngeal nerve. If both recurrent laryngeal nerves were severed during an operation such as a thyroidectomy, you would expect severe airway obstruction.

Ten hours after total thyroidectomy, a patient is found to be obtunded and cyanotic. The patient's SpO2 is 70%. What is the MOST likely etiology of this clinical deterioration? Acute hypocalcemia Bilateral recurrent laryngeal nerve injury Postoperative hematoma Tracheomalacia

Postoperative hematoma A compressive hematoma is the most common cause of airway obstruction 24 hours after thyroidectomy. Acute hypocalcemia typically manifests 24 to 48 hours postoperatively with laryngeal stridor and airway obstruction. Prior to stridor, the patient may complain of a tingling sensation in the lips and fingers. If recurrent laryngeal nerve damage occurs, it is more likely to be unilateral and present with hoarseness. If both recurrent laryngeal nerves were severed, severe airway obstruction occurs immediately. Hematoma is the most common cause of airway obstruction within 24 hours of a thyroidectomy. The definitive treatment of a hematoma is opening the surgical incision to evacuate the hematoma and relieve the airway obstruction.

A 3-month-old infant presents to the hospital febrile, lethargic, and tachypneic. Upon questioning, the infant's parents indicate that the infant had been treated for colic with a combination of aspirin and a bismuth subsalicylate solution for the past several weeks. Salicylate toxicity is suspected. An arterial blood gas would MOST likely show which of the following abnormalities? Primary metabolic acidosis with respiratory compensation Primary metabolic alkalosis with respiratory compensation Primary respiratory acidosis with metabolic compensation Primary respiratory alkalosis with metabolic compensation

Primary metabolic acidosis with respiratory compensation This infant is suffering from severe salicylate toxicity. Patients taking large doses of bismuth subsalicylate solutions often do not realize that these drugs contain high concentrations of salicylate. Bismuth subsalicylate has an aspirin equivalency conversion factor of 0.479 (approximately half the strength of aspirin). Salicylate toxicity occurs as a result of salicylate's interference with both Kreb's cycle enzymes and, at high level, uncoupling of oxidative phosphorylation. This results in a primary metabolic acidosis, to which most patients will develop a respiratory compensation.

A 63-year-old woman with a six-month history of headache and visual disturbances is diagnosed with a non-functioning pituitary macroadenoma. Serum levels of which following hormones is MOST likely to be elevated in this patient? Adrenocorticotropic hormone (ACTH) Growth hormone (GH) Prolactin Thyroid-stimulating hormone (TSH)

Prolactin The mass effects of an expanding pituitary tumor are well-characterized. Many patients complain of a headache. Patients with macroadenomas may complain of visual loss (classically temporal or bitemporal hemianopsia) from compression of the optic chiasm. Compression of the normal pituitary gland by the tumor can cause anterior pituitary compression and dysfunction, resulting in hypopituitarism and low levels of ACTH, TSH, GH, luteinizing hormone (LH), and follicle stimulating hormone (FSH). Patients may become hyperprolactinemic secondary to a loss of tonic inhibition of prolactin secretion.

An 85-year-old man is in atrial fibrillation in the intensive care unit after aortic valve replacement. Amiodarone 150 mg is administered by intravenous bolus injection. Which of the following BEST describes that mechanism of amiodarone? Blocks cardiac sodium channels Prolongs repolarization Shortens the QT interval Slows conduction at the AV node

Prolongs repolarization Amiodarone is a Class III antiarrhythmic agent and is used to treat both atrial and ventricular arrhythmias. It prolongs repolarization and lengthens the cardiac action potential, provides negative chronotropy in nodal tissue, blocks cardiac potassium and calcium channels. This slows conduction at the SA node. It does also have a negative inotropic effect and causes significant peripheral vasodilation. The two main side effects from administration of amiodarone are bradycardia and hypotension. These can be lessened by slowing the rate of administration, providing a fluid bolus, supporting with a vasopressor during the infusion or temporary pacing if the bradycardia is profound.

Small amounts of opioids are often used to supplement the analgesic and anesthetic effects of local anesthetics administered in the epidural space. In the obstetric patient, which of the following is the MOST common side effect of 100 mcg of epidural fentanyl? Difficulty "pushing" if administered too close to delivery Pruritus Respiratory depression Uterine atony

Pruritus Intrathecal fentanyl results in profound visceral pain relief. As fentanyl is highly lipophilic, it rapidly leaves cerebrospinal fluid and penetrates the spinal cord as well as the systemic circulation. In contrast to local anesthetics, fentanyl does not contribute to motor block or difficulty pushing. Unlike the more hydrophilic morphine, fentanyl does not have significant rostral spread within the intrathecal space and is therefore not thought to contribute to maternal respiratory depression. Common side effects of intrathecal fentanyl include pruritus (especially of the nose and trunk) and nausea, although pruritus is the most common. Epidural administration of fentanyl also results in significant systemic absorption. It can cross the placenta and result in a transient decrease in fetal heart rate variability, which can make interpretation of fetal heart rate patterns challenging. It is not thought to contribute to newborn respiratory depression under normal circumstances.

A 58-year-old man is admitted with chest pain, ST elevation in ECG leads II, III and aVF, and increased cardiac troponin levels. His echocardiogram shows regional wall motion abnormalities in the inferior wall. Which of the following medications should be initiated within 24 hours after admission to improve his survival following his myocardial infarction? Hydrochlorothiazide Losartan Metoprolol Ramipril

Ramipril For patients who have sustained a myocardial infarction, there are numerous studies that show a survival benefit for starting angiotensin converting enzyme (ACE) inhibitor medications within 24 hours after MI. Beta blockers are important in care of the patient with myocardial ischemia in order to reduce heart rate (decrease demand and increase supply). ACE inhibitors and angiotensin receptor blockers work by inhibiting the renin-angiotensin aldosterone system and are theorized to help prevent or slow ventricular remodeling after MI, which allows for improvement in ejection fraction.

A patient with a cerebral tumor and seizure disorder is being treated with phenobarbital for seizure prophylaxis. Which of the following is the MOST likely anesthetic implication of chronic phenobarbital? Etomidate is contraindicated Propofol is contraindicated Slower metabolism of hepatically metabolized drugs Rapid metabolism of hepatically metabolized drugs

Rapid metabolism of hepatically metabolized drugs Phenobarbital, primidone (which is converted to phenobarbital), phenytoin, and carbamazepine are potent hepatic enzyme inducers. The most noticeable effect is the short duration of clinical effect with the aminosteroid neuromuscular blocking agents (i.e., vecuronium, rocuronium).

A 67-year-old, 96-kg man underwent ultrasound-guided supraclavicular blockade for a 2-hour right wrist surgery. Forty ml of 0.5% bupivacaine was injected under ultrasound guidance. In the recovery room the patient reports that his breathing does not feel "normal" and you notice that his pupil diameter is unequal with the right pupil being smaller. Vital signs include HR 65 bpm, BP 117/68 mmHg, SpO2 98% on room air. Which of the following is the BEST course of action? Initiate an intralipid infusion Obtain an upright Chest X-Ray Perform an emergent needle decompression of the right lung Reassure the patient

Reassure the patient Although pneumothorax is possible during ultrasound supraclavicular blockade, it is a very rare occurrence. With traditional landmark based supraclavicular block, it is estimated that pneumothorax occurred in 0.5% to 5%. The most common side effects during supraclavicular block are Horner's syndrome (ipsilateral eye ptosis, miosis and anhidrosis) and phrenic nerve blockade. These occur less frequently compared to an interscalene nerve block and are estimated to occur in 30% to 50% of supraclavicular blocks and are more likely when local anesthetic volumes >20 cc are used. Reassure the patient that the symptoms will resolve as the block resolves. This is clearly not local anesthetic toxicity and therefore intralipid infusion would not be helpful.

Which of the following factors is LEAST likely to be associated with uterine atony? Infection Polyhydramnios Recent ketorolac Tocolytic therapy

Recent ketorolac Uterine atony is not only the leading cause of postpartum hemorrhage, but it is also the most common indication for a peripartum blood transfusion. Postpartum hemostasis involves the release of endogenous uterotonic factors, and atony results when there is failure of adequate uterus contraction after delivery. This can occur for various reasons including pre-delivery use of oxytocin (via receptor down-regulation), over-distension of the uterus due to polyhydramnios or multiple gestations, and chorioamnionitis. Tocolytic therapy relaxes uterine muscle, making it more difficult to contract after delivery. Although non-steroidal anti-inflammatory drugs can inhibit platelet function, they have minimal effect on uterine muscle tone.

Which nerve innervates the only abductor muscle of the vocal cords? Internal branch of superior laryngeal nerve Recurrent laryngeal nerve External branch of superior laryngeal nerve Glossopharyngeal nerve

Recurrent laryngeal nerve The only abductor of the vocal cords is the posterior cricoarytenoid muscle. All muscles of the larynx apart from the cricothyroid muscle are innervated by the recurrent laryngeal nerve. Therefore, the only abductor muscle of the vocal cords is innervated by the recurrent laryngeal nerve. If both recurrent laryngeal nerves were severed during an operation such as a thyroidectomy, you would expect severe airway obstruction.

Which nerve provides sensory innervation to the airway below the level of the vocal cords? External branch of superior laryngeal nerve Internal branch of superior laryngeal nerve Glossopharyngeal nerve Recurrent laryngeal nerve

Recurrent laryngeal nerve The recurrent laryngeal nerve provides sensory innervation to the larynx below the level of the vocal cords. The recurrent laryngeal nerve is a branch of the vagus nerve (CN X). The internal branch of the superior laryngeal nerve provides sensory innervation to the larynx above the level of the vocal cords. The superior laryngeal nerve is also a branch of the vagus nerve. The external branch of superior laryngeal nerve provides motor innervation to the cricothyroid muscle. The glossopharyngeal nerve provides sensory innervation to the pharynx.

A 55-year-old man is admitted with progressively worsening headache. Non-contrast head CT suggests cerebral venous sinus thrombosis and the diagnosis is confirmed by CT venography. He is treated with intravenous heparin but becomes more somnolent. Which of the following is the MOST appropriate management at this time? Add argatroban (direct thrombin inhibitor) Repeat head CT and send for endovascular therapy Add a glycoprotein IIb/IIIa inhibitor Increase heparin infusion to target a higher PTT

Repeat head CT and send for endovascular therapy This patient has progressive neurologic decline despite therapeutic anticoagulation. Venous infarctions, which can frequently become hemorrhagic, are common. Repeating the head CT to rule out intra-cerebral hemorrhage (most commonly from venous infarction) and/or large ischemic infarction is the most appropriate immediate step. There is no high level evidence to guide subsequent therapeutic decisions. Most neurointensivists and neurointerventionalists would opt for endovascular therapy (catheter directed thrombolysis and/or mechanical clot disruption) in this setting.

Regarding roller pumps and centrifugal pumps utilized for cardiopulmonary bypass, which of the following statements is MOST correct? Retrograde flow is possible with the roller pump Retrograde flow is possible with the centrifugal pump Retrograde flow is not possible with either the roller pump nor the centrifugal pump Retrograde flow is possible with either the roller pump or the centrifugal pump

Retrograde flow is possible with the centrifugal pump The centrifugal pump is quite different than the roller pump. The centrifugal pump operates on a principle of a constrained vortex, where a rotator (impeller) is housed within a rigid container shaped like a cone. Flow depends on the pressure differential created by spinning cones within the pump. In other words, rapidly rotating cones create negative pressure (pressure drop) by the centrifugal action of the rotating core, propelling fluid forward. Flow varies depending on pump preload and afterload. Unfortunately, retrograde flow is possible, but of course, safety checks are in place to prevent this.

A 59-year-old man presents with nausea, diaphoresis, and bradycardia. His blood pressure is 80/40 mmHg, HR 45 bpm. A transthoracic echocardiogram reveals global left ventricular hypokinesis with bulging of the interventricular septum into the left ventricle, mild mitral regurgitation, and moderate tricuspid regurgitation. Which coronary artery do you MOST expect to be occluded? First diagonal artery Left anterior descending artery Left circumflex artery Right coronary artery

Right coronary artery The symptoms of this patient as well as the echocardiogram findings indicate right ventricular ischemia. Patients typically often present with symptoms, such as nausea, vomiting, diaphoresis and bradycardia, and may not have the typical chest pain or pressure symptoms. An inferior wall myocardial infarction (right coronary artery distribution in 85% of patients) may manifest as complete heart block due to damage to the AV node. The SA node is fed by the RCA in 55% of population and left circumflex artery in 45% of population.

An otherwise healthy 61-year-old woman is in the operating room for the placement of an intramedullary rod after a femur fracture 2 days ago. During reaming, her SpO2 falls from 98% to 85% on FiO2 0.5, she develops sinus tachycardia to 120 bpm and her blood pressure is currently 65/40 mmHg. Which of the following would you MOST expect to see on an intraoperative transesophageal echocardiogram? Anterolateral regional wall dyskinesis Global left ventricular hypokinesis Patent foramen ovale Right ventricular dilation

Right ventricular dilation A history of trauma to long bones or an orthopedic procedure with intramedullary reaming of the bone may result in fat embolism syndrome. In a patient under general anesthesia, the signs are few but include hypoxia, increased A-a gradient, tachycardia, and a petechial rash on the upper body. If the fat embolism is large enough, fat, blood components and bony particles travel via the venous system to the right side of the heart and become lodged in the pulmonary arteries, causing increased afterload against which the right ventricle must pump. The right ventricle acutely dilates and fails as it is accustomed to performing low pressure, volume work. This causes the interventricular septum to bulge into the left ventricle and reduce the stroke volume being pumped to the systemic circulation (thus reducing arterial systemic blood pressure). It is not uncommon, if a patient has a patent foramen ovale (PFO), for debris to cross from right to left via the opening, but a PFO is not specific to fat embolism syndrome. Hypokinesis and dyskinesis of parts of the left ventricle occur with myocardial ischemia, which is not usually part of the initial response to fat embolism.

Regarding cardiopulmonary bypass, if pump outflow is occluded, excessive pressure can build proximal to the occlusion if which kind of pump is used? Roller pump Centrifugal pump Neither roller nor centrifugal pump Either roller or centrifugal pump

Roller pump During cardiopulmonary bypass, a mechanical pump is required to circulate blood through the circuit and then back to the patient. In general, 2 types of pumps are utilized for this purpose: roller pumps and centrifugal pumps. Flow of a roller pump is predictable and depends on the revolutions per minute of the pump. Although retrograde flow is not possible, if there is outflow occlusion to the pump, excessive pressure can build, causing the tubing to rupture or the tubing connections to separate. Of course, there are safety checks in place to prevent this from occurring.

Which of the following has NOT been definitely shown to reduce central line infection rates? Chlorhexidine-impregnated sponges Routine dressing changes Use of a checklist Use of single lumen catheters

Routine dressing changes A 2x2 factorial randomized controlled trial comparing chlorhexidine-impregnated sponges and less frequent dressing changes to traditional management of intravascular catheters included these sponges on the arterial catheters, and showed that chlorhexidine-impregnated sponges on both the arterial and central venous catheters lowers CRBSI from 1.4:1000 to 0.6:1000 catheter days. Implementation of a central line checklist has also been shown to be extremely efficacious at reducing the incidence of CRBSI. Silver-impregnated subcutaneous cuffs and routine, scheduled catheter site changes are not supported by any meaningful data. Single lumen catheters are associated with a lower infection rate and should be used when clinically feasible.

Which of the following cardiac findings during pregnancy is MOST ABNORMAL? Cardiomegaly on chest x-ray Right bundle branch block S4 heart sound Tricuspid regurgitation

S4 heart sound Cardiovascular changes in pregnancy revolve around the themes of increasing cardiac output and extension of blood flow to the placenta. Heart rate and stroke volume increase, while systemic vascular resistance decreases to reduce afterload and maintain a normal blood pressure. As a result of the increased volumes occupying the heart, a mild but normal increase in heart size can be seen and a third heart sound (S3) can be heard. Dilation of the heart can also cause new regurgitant murmurs, particularly tricuspid regurgitation, with a resulting systolic murmur. The dilation can temporarily alter conduction in the heart, leading to right-axis deviation and right bundle branch block. An S4 heart sound is almost always pathologic, and this holds true in pregnant women: it should be investigated fully if present.

Four days after an ureteral stent placement, an 87-year-old woman becomes increasingly confused. Vital signs include T 38.9°C, BP 100/60 mmHg (MAP 73), HR 103 bpm, respiratory rate 16 /min, SpO2 96% on room air. White blood cell count is 16,000 µL-1, creatinine is 2.2 mg/dL (up from 0.8 mg/dL baseline), and lactate is 3 mmol/L. Urinalysis shows leukocytes and positive nitrite; a urine culture is pending. Which of the following is this patient's MOST appropriate sepsis classification? Sepsis Severe sepsis Sepsis-induced hypotension Septic shock

Severe sepsis According to the 2012 International Guidelines for Management of Severe Sepsis and Septic Shock, sepsis is defined as the probable or known presence of an infection together with systemic manifestations of infection. Severe sepsis is defined as sepsis plus sepsis-induced organ dysfunction or tissue hypoperfusion. Sepsis-induced hypotension is defined as a systolic blood pressure < 90 mmHg or mean arterial pressure < 70 mmHg or systolic blood pressure decrease > 40 mmHg or less than two standard deviations below normal for age in the absence of other causes of hypotension. Septic shock is sepsis-induced hypotension that persists despite adequate fluid resuscitation. Any of the following signs of organ dysfunction or tissue hypoperfusion thought to be due to the infection would lead to a severe sepsis classification: Sepsis-induced hypotension Lactate above upper limits laboratory normal Urine output < 0.5 mL/kg/hr for more than 2 hrs despite adequate fluid resuscitation Acute lung injury with Pao2/Fio2 < 250 in the absence of pneumonia as infection source Acute lung injury with Pao2/Fio2 < 200 in the presence of pneumonia as infection source Creatinine > 2.0 mg/dL (176.8 μmol/L) Bilirubin > 2 mg/dL (34.2 μmol/L) Platelet count < 100,000 μL Coagulopathy (international normalized ratio > 1.5)

A 79-year-old patient suffers an ascending aortic dissection and presents with acute pericardial tamponade physiology. The patient demonstrates pulsus paradoxus on his arterial waveform tracing. What physiologic event MOST accounts for this pulsus paradoxus? Increased systemic vascular resistance Reduced diastolic filling Shift of interventricular septum Tachycardia

Shift of interventricular septum In acute tamponade physiology, the pericardial sac does not have time to stretch to accept increased fluid thus the effusion reaches a critical volume early and causes reduction in filling of the cardiac chambers with increased intracavitary pressures. This drastically reduces preload and cardiac output. Venous return is normally biphasic but becomes confined to systole in severe tamponade and ceases during diastole when intrapericardial pressures are maximal. During inspiration there is increased right-sided filling, which causes the interventricular septum to shift toward the left side of the heart. This causes reduction in blood pressure during inspiration and the septum shifts back during expiration and thus blood pressure goes up during the expiratory phase.

Which of the following causes of hypoxemia will be LEAST likely to improve with increasing FiO2? Hypoventilation Impaired diffusion Shunt Ventilation-perfusion mismatch

Shunt Arterial hypoxemia results from absolute shunt despite oxygen supplementation. Other causes of hypoxemia, including hypoventilation, ventilation-perfusion mismatch, and impaired diffusion may show improvement with increasing supplemental oxygen.

Local anesthetics block which of the following channels for their primary action? Sodium Calcium Potassium Magnesium

Sodium Local anesthetics act by inhibiting sodium influx through voltage-gated sodium channels within neuronal cell membranes. This influx of sodium prevents an action potential and blocks conduction.

Which of the following is NOT an absolute contraindications to Ramipril? Hereditary angioedema Pregnancy Renal artery stenosis Sulfonamide allergy

Sulfonamide allergy Ramipril is an ACE-inhibitor (ACE-I). ACE-Is are contraindicated in patients with a history of angioneurotic edema, even if angioedema is not due to an ACE-I or C1 esterase deficiency. The use of ACE-Is during pregnancy is associated with serious complications in the fetus, including congenital malformations and intrauterine fetal demise. Patients with renal artery stenosis can develop serious hypotension when treated with an ACE-I. While captopril contains a sulfhydryl group, there is not thought to be cross-reactivity with sulfonamides. Regardless, ramipril may be safely administered to patients with a history of allergy to sulfonamides.

Which of the following positions is associated with the LOWEST risk of ventilator associated pneumonia? Prone Supine head up Supine head down Supine flat

Supine head up In a retrospective review of 109 mechanically ventilated patients, Kollef found an increased risk of pneumonia in supine patients (adjusted odds ratio 2.9). A randomized trial of 50 patients suggested that gastroesophageal reflux (detected with scintigraphy) was more likely in supine patients as compared to semi-recumbent. Another randomized controlled trial of semi-recumbent versus supine positioning in mechanically ventilated patients was stopped early because of an increased risk of both clinically suspected (34% vs. 8%, p = 0.003) and microbiologically confirmed (23% vs. 5%, p = 0.016) pneumonia in supine patients.

An infant had a patent ductus arteriosus closed and is recovering in the NICU. You note that the patient has new-onset stridor and hoarseness. Which of the following is the MOST likely cause of this finding? Surgical dissection causing recurrent laryngeal nerve injury Hypocalcemia after transfusion of packed red blood cells Laryngospasm after deep extubation Incorrectly sized endotracheal tube

Surgical dissection causing recurrent laryngeal nerve injury The incidence of recurrent laryngeal nerve injury with standard PDA ligation is reported to be 4.2% by Fan et al. while other authors indicate 2.5% using VATS clipping. Symptoms attributable to vocal cord paralysis regress in most cases and usually less than 1% of patients have lasting dysfunction. Clip entrapment of the nerve has been cited to be the mechanism of injury, but it may be that trauma induced by traction (or thermal injury by electrocautery) may better explain the observed clinical outcome. Zbar et al. reports a series of PDAs treated using open thoracotomy and indicates an incidence of recurrent laryngeal nerve injury of 22.7% in extremely low-weight babies, confirming the importance of the issue in premature infants. Decreased incidence of this complication appears to have been achieved with VATS and may be a consequence of improved vision from the video camera image.

At which point of the ECG cycle should an intraaortic balloon pump (IABP) inflate? P wave Q wave R wave T wave

T wave For an intraaortic balloon pump (IABP) to have optimal effect, the inflation and deflation must be timed appropriately to the patient's cardiac cycle. This may be done using the ECG tracing or the arterial waveform. The IABP is triggered from the R wave of the ECG signal but the balloon inflation begins in the middle of the T wave, with deflation prior to the end of the QRS complex. Thus the balloon should inflate at the beginning of diastole (noted as a second diastolic augmentation at the position of the dicrotic notch on the arterial waveform). The balloon deflates prior the arterial upstroke. If the IABP is mistimed, it can have significant deleterious effects on the hemodynamics, such that if the balloon inflates during systole, the left ventricle will not be able to empty against a high afterload and may worsen the heart failure.

At approximately what level or above will a patient with spinal cord injury be at GREATEST risk for autonomic hyperreflexia? C8 T4 T7 T10

T7 A spinal cord injury at or above T7 (T5-T8, depending on reference) predisposes a patient to autonomic hyperreflexia. Autonomic hyperreflexia is typically first seen within four to six months of spinal cord injury and can continue to occur for years. With a stimulus below the level of the spinal cord injury lesion, intact lower motor neurons send an impulse up the spinal cord. However, this impulse is interrupted at the site of the spinal cord injury and unable to reach the cerebral cortex. This leads to an impaired feedback loop. However, the ascending signal reaches the thoracic sympathetic splanchnic nerves resulting in hypertension. This hypertension is recognized by the carotid sinus and aortic arch baroreceptors leading to a parasympathetic reflexive bradycardic response via the vagus nerve. No autonomic nervous system changes occur below the level of spinal cord injury since these signal pathways are also interrupted.

The arteria radicularis magna, or artery of Adamkiewicz, most commonly arises from: T4 - T8 T8 - L2 L2 - L4 L4 - S1

T8 - L2 A major complication of thoracic aortic surgery is paraplegia, occurring in up to 20% of elective cases, and is secondary to spinal cord ischemia. The arteria radicularis magna supplies blood to the anterior spinal artery. The arteria radicularis magna has a variable origin from aorta, arising between T5 - T8 in 15%, between T9 - T12 in 60% and between L1 - L2 in 25% of individuals.

The criteria for systemic inflammatory response syndrome includes which of the following parameters? Temperature, blood pressure, respiratory rate or PaCO2, white blood cell count Temperature, heart rate, respiratory rate or PaCO2, urine output Temperature, heart rate, respiratory rate or PaCO2, white blood cell count Temperature, blood pressure, respiratory rate or PaCO2, urine output

Temperature, heart rate, respiratory rate or PaCO2, white blood cell count The systemic inflammatory response syndrome is defined by the presence of 2 or more of the following parameters: body temperature greater than 38°C or less than 36°C, heart rate greater than 90 bpm, respiratory rate greater than 20 breaths per min or PaCO2 less than 32 mmHg, and leukocytosis greater than 12,000/mm3 or less than 4000/mm3.

The elimination half-life of intravenously administered oxytocin in the parturient is approximately: 30 to 120 seconds 3 to 5 minutes 10 to 15 minutes 20 to 30 minutes

The elimination half-life of intravenously administered oxytocin in the parturient is approximately: 3 to 5 minutes Both endogenous and intravenously administered oxytocin have short elimination half-lives of about 3 to 5 minutes. As a result, oxytocin must be administered as a continuous infusion for the induction of labor.

The essential component of cardioplegia solutions is: mannitol magnesium potassium corticosteroid

The essential component of cardioplegia solutions is: potassium High concentrations of potassium (20 - 30 mEq/L) are used in cardioplegia solutions. These solutions result in an increase in extracellular potassium and reduce transmembrane potential. This progressively interferes with the normal sodium currents of depolarization and eventually the sodium channels are completely inactivated.

A 52-year-old man with a parathyroid adenoma presents for surgical resection. Which of the following ECG changes are MOST consistent with hypercalcemia? Peaked P-wave Peaked T-wave Short PR interval Short QT interval

The most common ECG change in hypercalcemia is a short QT interval. Peaked T-waves are most commonly encountered in hyperkalemia. A short PR interval may indicate a preexcitation syndrome via an accessory pathway that leads to early activation of the ventricles, such as Wolff-Parkinson-White syndrome. A large or peaked P-wave (>2.5mm) can be encountered in hypokalemia. A large P-wave can also indicate right atrial enlargement.

A 73-year-old man with prostate adenocarcinoma presents for robotic-assisted prostatectomy. His medical history is significant for poorly controlled, refractory hypertension, COPD, chronic kidney disease, and non-obstructive coronary artery disease. His home medications include aspirin, hydrochlorothiazide, lisinopril, and amlodipine. His blood pressure in the pre-op holding area is 177/98 mmHg. Anticipating likely intraoperative hypertension, you consider the vasodilatory agents in your armamentarium. Which of the following is MOST correct regarding these drugs? The vasodilatory effects of nitroglycerin on the arterial and venous sides are similar The pharmacologic half-life of hydralazine is prolonged in patients with chronic kidney disease Nesiritide is less effective than nitroglycerin in reducing right atrial pressure The most common acid-base disturbance seen with sodium nitroprusside overuse is metabolic acidosis

The most common acid-base disturbance seen with sodium nitroprusside overuse is metabolic acidosis Nitroglycerin is a much more potent venodilator than arterial dilator. Hydralazine is metabolized by acetylation in the liver, so an increase in the half-life would not be expected in a patient with chronic kidney disease. According to Barash, nesiritide is even more effective than nitroglycerin in reducing right atrial pressure. Metabolic acidosis is part of the triad of cyanide toxicity (along with tachyphylaxis and elevated SvO2) that can be seen with sodium nitroprusside administration.

You are called to intubate a patient in the emergency department who was found pulseless in front of the hospital. Which of the following is MOST correct regarding the use of cricoid pressure during endotracheal intubation in this scenario? Cricoid pressure should be used only if the victim is unconscious The routine use of cricoid pressure is not recommended Cricoid pressure should be used if the patient has food contents in mouth at the time of intubation Cricoid pressure should only be applied by trained personnel

The routine use of cricoid pressure is not recommended Per the 2010 ACLS Guidelines: "...Cricoid pressure can prevent gastric inflation and reduce the risk of regurgitation and aspiration during bag-mask ventilation, but it may also impede ventilation. Seven randomized studies showed that cricoid pressure can delay or prevent the placement of an advanced airway and that some aspiration can still occur despite application of cricoid pressure. In addition, it is difficult to appropriately train rescuers in use of the maneuver. Therefore, the routine use of cricoid pressure in cardiac arrest is not recommended."

Which of the following is NOT a contraindication to activated charcoal in the setting of suspected aspirin poisoning? Bowel obstruction Bowel perforation Inability to protect airway Two hours since ingestion

Two hours since ingestion

All of the following are appropriate landmarks for sizing an oropharyngeal airway except the: Tip of nose Corner of patient's mouth Earlobe Angle of mandible

Tip of nose

Which of the following is LEAST consistent with a Phase II block after succinylcholine? Fade with tetanic stimulation Post-tetanic potentiation Train-of-four ratio of 1.0 Total dose of succinylcholine >4 mg/kg

Train-of-four ratio of 1.0 A Phase II block is typically encountered after a large dose of succinylcholine (i.e., a large single dose, repeated doses, or a continuous infusion) where the total amount is greater than 4 mg/kg. Repeated stimulus of the acetylcholine receptor results in desensitization at the nerve terminal and the myocyte becomes less sensitive to acetylcholine, with prolonged neuromuscular blockade. Thus, with nerve stimulation, a Phase II block will have the features of a non-depolarizing block: Fade with tetanic stimulation; Post-tetanic potentiation; and a Train-of-four ratio of less than 0.4.

Which of the following is LEAST likely to occur following a celiac plexus block? Diarrhea Orthostatic hypotension Retroperitoneal hemorrhage Urinary incontinence

Urinary incontinence The celiac plexus contains preganglionic sympathetic fibers from greater and lesser splanchnic nerves and postganglionic sympathetic and preganglionic parasympathetic fibers. It provides sensory innervation and sympathetic outflow to the stomach, liver, spleen, pancreas, kidney, and GI tract up to splenic flexure. Thus, blockade of the celiac plexus results in a loss of sympathetic innervation. The loss of sympathetic outflow can result in both diarrhea and orthostatic hypotension. During performance of the block, inadvertent puncture of the aorta or vena cava can result in a retroperitoneal hemorrhage; however, this is less common. Urinary incontinence does not occur and the urinary tract is not innervated by the celiac plexus.

Prior to pneumonectomy, split lung function testing is indicated in the patient with: an FEV1 of 2.2 L a PaCO2 of 49 mm Hg on room air a PaO2 of 54 mm Hg on room air a maximum VO2 of 21 mL/kg/min

a PaCO2 of 49 mm Hg on room air - Split lung function testing is indicated in patients requiring pneumonectomy, but not meeting the recommended laboratory criteria. Current recommendations for patients requiring pneumonectomy are: PaCO2 < 45 mm Hg FEV1 > 2 L Predicted postop FEV1 > 800 mL Maximum VO2 > 15 mL/kg/min FEV1/FVC > 50% of predicted

The effects of 60-Hz current on a human for 1-second match the current to the physiologic effect. a.) Perception Threshold b.) Let-Go Current c.) Microshock Risk d.) V Fib Choices: 100 micro amps; 200 mAmp; 15 mAmp; 1 mAmp

a.) 1 mAmp b.) 15 mAmp c.) 100 micro-amps d.) 200 mAmp

Renal blood flow: (Select 2) is largely determined by renal oxygen consumption accounts for 20 - 25% of the cardiac output is distributed mostly to juxtamedullary nephrons can be directed away from cortical nephrons by sympathetic stimulation is not autoregulated

accounts for 20 - 25% of the cardiac output, can be directed away from cortical nephrons by sympathetic stimulation The kidneys are the only organ for which oxygen consumption is determined by blood flow; the reverse is true in other organs. The kidneys receive 20 - 25% of the cardiac output with only 10 - 15% going to the juxtamedullary nephrons and 80% going to cortical nephrons. However, blood flow can be redirected to juxtamedullary nephrons by increased levels of catecholamines and angiotensin II. Autoregulation of RBF occurs between mean arterial pressures of 80 - 180 mm Hg.

The single greatest cause of mortality in the patient with sickle cell disease is a result of: acute chest syndrome sequestration crisis aplastic crisis vaso-occlusive crisis

acute chest syndrome - ACS represents the single greatest threat to the patient with SCD as the mortality is 1% to 20%. The diagnosis of ACS can be made when there are new lung infiltrates on a chest radiograph in the presence of any of the following: chest pain, cough, dyspnea, wheezing, or hypoxemia. Proposed mechanisms of ACS are thrombosis, embolism (clot and fat), and infection. The frequency of ACS after abdominal surgery is 10% to 20%.

Carbonic anhydrase inhibitors are used in the treatment of: acute glaucoma renal tubular acidosis diarrhea induced acidosis acidosis resulting from hypoventilation

acute glaucoma Carbonic anhydrase inhibitors decrease the ability of the kidneys to reabsorb bicarbonate, resulting a hyperchloremic acidosis. As a result, carbonic anhydrase inhibitors would be avoided in patients with acidosis, especially a normal-anionic-gap acidosis. Because bicarbonate is filtered by the ciliary process in the formation of aqueous humor, carbonic anhydrase inhibitors reduce the formation of aqueous humor and can be used to decrease intraocular pressure.

Safety features that prevent filling of the vaporizer with an incorrect agent include: the pin index safety system agent-specific keyed filling ports the diameter index safety system counter-threading of the bottle attachment

agent-specific keyed filling ports -Modern vaporizers offer agent-specific keyed filling ports to prevent filling with an incorrect agent. The pin-index safety system is found on e-cylinders to prevent incorrect tank placement. M&M pg 63

The perception of an ordinarily non-noxious stimulus as pain is referred to as: allodynia anesthesia dolorosa dysesthesia hyperalgesia

allodynia - Allodynia is the perception of non-noxious stimuli as pain. Dysesthesia is an unpleasant sensation without a stimulus. Hyperesthesia is an increased response to a mild stimulus. Anesthesia dolorosa is pain in an area that lacks sensation. - Barash pg. 1649-1650

Correct statements concerning the use of antidepressants in pain management include: - analgesic effects require a higher dose than that needed for antidepression - analgesic effects appear to be secondary to the blockade of serotonin and norepinephrine reuptake - antidepressants are not effective in neuropathic pain - newer SSRIs are more effective analgesics than the older tricyclic antidepressants

analgesic effects appear to be secondary to the blockade of serotonin and norepinephrine reuptake Antidepressants demonstrate an analgesic effect at doses lower that those needed for antidepressant effect. Both actions appear secondary to the block of the reuptake of serotonin and norepinephrine. Older tricyclic antidepressants seem more effective analgesics than the newer SSRIs. Antidepressants are most useful in patients with neuropathic pain.

Release of aldosterone by the adrenal cortex is stimulated by: (select 3) angiotensin I angiotensin II hypokalemia pituitary ACTH congestive heart failure hypervolemia

angiotensin II, pituitary ACTH, congestive heart failure Aldosterone release is stimulated by the renin-angiotensin system, but specifically by angiotensin II. Other causes of aldosterone release include hyperkalemia, ACTH release, hypovolemia, hypotension, CHF and the stress response.

As compared to hygroscopic heat-moisture exchangers, hydrophobic heat-moisture exchangers: - depend on electrostatic properties to retain moisture - are more effective filters of pathogens - are more efficient devices for retaining heat and moisture - are more likely to become saturated and increase resistance to ventilation

are more effective filters of pathogens - HMEs may be either hygroscopic or hydrophobic. Hygroscopic HMEs use paper or other fiber barrier, which possess electrostatic properties. They adsorb water in expiration and release it in inspiration and are efficient devices for retaining heat and moisture. Being fiber based, they are more prone to becoming saturated and causing increased resistance to ventilation. Hydrophobic HMEs are more efficient filters of pathogens, but are unable to deliver the degree of humidification offered by hygroscopic HMEs.

The most consistent clinical manifestation of aspiration pneumonitis is: bronchospasm arterial hypoxemia pulmonary vasoconstriction tachypnea

arterial hypoxemia Inhaled gastric fluid is rapidly distributed throughout the lungs, leading to destruction of surfactant-producing cells, damage to the pulmonary capillary endothelium and resultant atelectasis and pulmonary edema. Arterial hypoxemia is the most consistent clinical finding associated with aspiration pneumonitis. Tachypnea, bronchospasm and pulmonary vasoconstriction with secondary pulmonary hypertension may also be present.

Characteristics of omphalocele include: (Select 2) location lateral to the umbilicus lacks a hernia sac results from the failure of midgut migration into the abdomen nitrous oxide should be used during the repair to ensure a rapid emergence association with trisomy 21 results from abnormal development of the right omphalomesenteric artery

association with trisomy 21, results from the failure of midgut migration into the abdomen Gastroschisis and omphalocele are characterized by defects in the abdominal wall that allow herniation of the viscera. Omphaoceles occur at the base of the umbilicus, have a hernia sac and are commonly associated with other anomalies. In contrast, gastroschisis is usually lateral to the umbilicus, lacks a hernia sac and is usually an isolated finding. Nitrous oxide is best avoided during repair as it may result in bowel distention making closure more difficult.

The highest level of protein binding is seen with: procaine lidocaine mepivacaine bupivacaine

bupivacaine The physicochemical property that determines the duration of action of a local anesthetic is lipid solubility, which is directly correlated with plasma protein binding. Bupivacaine and levobupivacaine have the highest degree of protein binding (97%).

Local anesthetic solutions that are isobaric with the cerebrospinal fluid include: (Select 2) tetracaine 0.5% in 5% dextrose bupivacaine 0.75% in normal saline procaine 10% in sterile water lidocaine 2% in normal saline bupivacaine 0.3% in sterile water lidocaine 5% in 7.5% dextrose

bupivacaine 0.75% in normal saline lidocaine 2% in normal saline

A decrease in cholinesterase activity has been associated with: obesity thyrotoxicosis alcoholism burns

burns - Burns, liver disease, 3rd trimester of pregnancy, carcinoma, renal failure and collagen diseases as well as certain drug therapy have been associated with a decrease in cholinesterase activity. Increased cholinesterase activity has been associated with obesity, alcoholism, thyrotoxicosis, nephrosis, psoriasis and electro-convulsive therapy.

Neuroleptic malignant syndrome: can be precipitated with the use of metoclopramide carries a mortality of over 80% can be treated with physostigmine administration can be diagnosed with muscle biopsy

can be precipitated with the use of metoclopramide Neuroleptic malignant syndrome is a rare complication of antipsychotic therapy. Meperidine and metoclopramide can also precipitate the disorder which appears to be secondary to dopamine blockade in the basal ganglia. The disease has many characteristics in common with MH including increased temperature, metabolic derangement and hyperthermia. The mortality is 20 - 30%. Treatment with dantrolene and dopamine agonist, bromocripitine, appears effective.

Autonomic hyperreflexia: is common with cord lesions below T8 can precipitate pulmonary edema is not effectively prevented by regional anesthesia can be prevented with adequate intraoperative sedation

can precipitate pulmonary edema Autonomic hyperreflexia should be suspected in patients with lesions above T5-8. Regional anesthesia and deep general anesthesia are effective in preventing autonomic hyperreflexia. Surgical stimulation in these patients without adequate anesthesia can result in pulmonary edema, myocardial ischemia and cerebral hemorrhage.

An action potential characterized by a spike followed by a plateau phase is seen in: peripheral sensory nerve cells peripheral motor nerve cells striated skeletal muscle cells cardiac muscle cells

cardiac muscle cells In contrast to the action potentials of nerve and skeletal muscle cells, the action potential of the cardiac myocyte is characterized by a sharp spike followed by a plateau phase (2), which results from the opening of slower calcium channels.

The formation of metanephrine is the result of: - catechol-O-methyltransferase metabolism of epinephrine - catechol-O-methyltransferase metabolism of norepinephrine - monamine oxidase metabolism of epinephrine - monamine oxidase metabolism of norepinephrine

catechol-O-methyltransferase metabolism of epinephrine - Catechol-O-methyltransferase (COMT) metabolizes epinephrine to metanephrine and norepinephrine to normetanephrine. Subsequently, monamine oxidase (MAO) further metabolizes metanephrine and normetanephrine to vanillymandelic acid (VMA).

The formation of metanephrine is the result of: - catechol-O-methyltransferase metabolism of epinephrine - catechol-O-methyltransferase metabolism of norepinephrine - monamine oxidase metabolism of epinephrine - monamine oxidase metabolism of norepinephrine

catechol-O-methyltransferase metabolism of epinephrine Catechol-O-methyltransferase (COMT) metabolizes epinephrine to metanephrine and norepinephrine to normetanephrine. Subsequently, monamine oxidase (MAO) further metabolizes metanephrine and normetanephrine to vanillymandelic acid (VMA).

The synthesis of acetylcholine from acetylcoenzyme A and choline is catalyzed by: free acetate anion choline acetyltransferase acetyl cholinesterase pseudocholinesterase

choline acetyltransferase The synthesis of acetylcholine occurs in the cholinergic nerve terminal. Acetyl Co-A and choline combine to form acetylcholine. This reaction is catalyzed by the enzyme choline acetyltransferase.

Closing capacity is defined as: closing volume + expiratory reserve volume functional residual capacity - residual volume closing volume + residual volume residual volume + expiratory reserve volume

closing volume + residual volume Closing capacity is the lung volume at which airways begin to close and is defined as the closing volume + residual volume.

Examples of Type IV hypersensitivity reactions include: contact dermatitis hemolytic transfusion reactions anaphylaxis angioedema

contact dermatitis Type IV hypersensitivity reactions are delayed and cell-mediated. Examples of Type IV reactions include contact dermatitis, tuberculin-type hypersensitivity and chronic hypersensitivity pneumonitis.

A fresh E-cylinder of oxygen: contains more liters of gas than an E-cylinder of nitrous oxide contains about 90% liquid oxygen and 10% oxygen as a gas contains about 660 liters of oxygen has a lower pressure than the pipeline oxygen supply

contains about 660 liters of oxygen A fresh E-cylinder of oxygen contains about 660 liters of oxygen and is pressurized to 1900 psi.

Parasympathetic preganglionic fibers are found in: (Select 3) cranial nerve IV cranial nerve VII cranial nerve IX cranial nerve XI thoracic nerve 9 thoracic nerve 11 sacral nerve 1 sacral nerve 2

cranial nerve VII, cranial nerve IX, sacral nerve 2 Parasympathetic preganglionic fibers are found in cranial nerves III, VII, IX and X as well as sacral nerves 2, 3 and 4.

Pulmonary effects of β2-adrenergic stimulation include: (Select 2) - inhibition of hypoxic pulmonary vasoconstriction - decreased bronchial secretions - pulmonary vasoconstriction - bronchodilation - redirection of blood flow to lower V/Q alveolar units - activation of type II pneumocytes

decreased bronchial secretions, bronchodilation - The tracheobronchial tree receives sympathetic innervation form the T1 - T4 nerve roots. β2 stimulation causes bronchodilation and decreased secretions. The sympathetic nervous system has minimal effects on pulmonary vascular tone. However, α1 stimulation causes some degree of pulmonary vasoconstriction.

Renal effects of nitrous oxide include: decreased renal blood flow secondary to decreased cardiac output decreased renal blood flow secondary to increased renal vascular resistance increased renal blood flow secondary to sympathetic stimulation increased glomerular filtration with increased reabsorption

decreased renal blood flow secondary to increased renal vascular resistance Nitrous oxide appears to decrease renal blood flow by increasing renal vascular resistance. This results in decreased glomerular filtration and decreased urine output.

During pregnancy, the minimum alveolar concentration (MAC): - decreases until the 20th week - increases until the 20th week - decreases throughout the pregnancy - increases throughout the pregnancy

decreases throughout the pregnancy The MAC progressively decreases during pregnancy, at term by as much as 40%. MAC returns to normal by the third day after delivery.

During pregnancy, the minimum alveolar concentration (MAC): decreases until the 20th week increases until the 20th week decreases throughout the pregnancy increases throughout the pregnancy

decreases throughout the pregnancy - The MAC progressively decreases during pregnancy, at term by as much as 40%. MAC returns to normal by the third day after delivery. - M&M pg. 826

Heart rate: is the sole determinant of cardiac output in the elderly is normally determined by the intrinsic rate of the AV node decreases with increasing age is increased by stimulation of the M2 cholinergic receptors

decreases with increasing age - Cardiac output is the product of stoke volume and heart rate. Heart rate is an intrinsic function of the SA node and decreases with increasing age. Enhanced vagal activity slows the heart via stimulation of the M2 cholinergic receptors.

Dextran 40 has been shown to improve microcirculation by: reducing blood density increasing blood density decreasing blood viscosity increasing blood viscosity

decreasing blood viscosity Dextran 40 has been shown to improve microcirculation presumably by decreasing blood viscosity thereby improving laminar flow in the microcirculatory beds. Both Dextran 40 and Dextran 70 possess antiplatelet effects and may interfere with blood typing.

Pancreatic somatostatin producing cells in the Islets of Langerhans are: alpha cells beta cells gamma cells delta cells

delta cells The Islets of Langerhans are comprised of four cell types: alpha cells producing glucagon, beta cells producing insulin, delta cells producing somatostatin and PP cells producing pancreatic polypeptide.

In patients receiving vecuronium, the greatest augmentation of neuromuscular blockade is seen with the use of: isoflurane sevoflurane desflurane nitrous oxide

desflurane -Volatile agents decrease the nondepolarizer dosage requirements. The degree of the augmentation of blockade depends on the inhalational agent, with desflurane > sevoflurane > isoflurane > nitrous oxide. -M&M pg. 213

Causes of normal-anion-gap acidosis include: renal failure starvation diarrhea lactic acidosis

diarrhea Normal-anion-gap acidosis is also called hyperchloremic acidosis and results from the selective loss of bicarbonate anion or the introduction of large amounts of chloride anion. Common causes include: diarrhea, hypoaldosteronism, renal tubular acidosis and increased intake of chloride containing acids sometimes found in hyperalimentation.

Pathophysiologic factors affecting the anesthetic management of patients with hypothyroidism include: hypernatremia hyperglycemia difficulty with intubation and airway management increased blood viscosity due to elevated hematocrit

difficulty with intubation and airway management - Potential problems of hypothyroidism include hypoglycemia, anemia, hyponatremia and difficulty during intubation because of a large tongue or the presence of a goiter. Hypothermia secondary to a low metabolic rate is a common postoperative complication. - Nagelhout pg 875-876

Characteristics of human immunodeficiency virus neuropathy include: (Select 2) distal polyneuropathy rapid sudden onset proximal muscle weakness allodynia upper extremities most commonly involved proximal to distal progression of symptoms

distal polyneuropathy allodynia - Symptomatic neuropathy occurs in 10% to 35% of patients who are seropositive for human immunodeficiency virus (HIV). The sensory neuropathies associated with HIV include distal sensory polyneuropathy and antiretroviral toxic neuropathy (ATN) secondary to the treatment. The clinical features of HIV sensory neuropathy typically include painful allodynia and hyperalgesia. The onset is gradual and most commonly involves the lower extremities. The neuropathy and dysesthesia progress from the distal to the more proximal structures. There is minimal subjective or objective motor involvement and this is generally limited to the intrinsic muscles of the foot.

Characteristics of human immunodeficiency virus neuropathy include: (Select 2) distal polyneuropathy rapid sudden onset proximal muscle weakness allodynia upper extremities most commonly involved proximal to distal progression of symptoms

distal polyneuropathy, allodynia Symptomatic neuropathy occurs in 10% to 35% of patients who are seropositive for human immunodeficiency virus (HIV). The sensory neuropathies associated with HIV include distal sensory polyneuropathy and antiretroviral toxic neuropathy (ATN) secondary to the treatment. The clinical features of HIV sensory neuropathy typically include painful allodynia and hyperalgesia. The onset is gradual and most commonly involves the lower extremities. The neuropathy and dysesthesia progress from the distal to the more proximal structures. There is minimal subjective or objective motor involvement and this is generally limited to the intrinsic muscles of the foot

Signs of cardiac tamponade include: (Select 2) distended neck veins increased QRS voltage seen on ECG decreased central venous pressure bradycardia systemic vasoconstriction an increase in systolic blood pressure during inspiration

distended neck veins, systemic vasoconstriction Cardiac tamponade is indicated by the presence of neck vein distention, hypotension, muffled heart sounds (Beck's triad) and a greater than 10 mm Hg decline in blood pressure during spontaneous inspiration (pulsus paradoxus). Tachycardia and systemic vasoconstriction are present to maintain blood pressure with the associated decreased stroke volume.

During mediastinoscopy the risk of air embolization is greatest: when the patient is supine during spontaneous ventilation immediately after closure of the incision in the postoperative period

during spontaneous ventilation Air embolization is seen with mediastinoscopy as a result of the 30o elevation of the head. This risk is increased if the patient is spontaneously ventilating, secondary to the negative intrathoracic pressures generated during inhalation.

Anesthetic implications of multiple sclerosis include: - exacerbation induced by spinal anesthesia - exacerbation induced by epidural anesthesia - exacerbation of symptoms secondary to hypothermia - the presence of significant peripheral neuropathy causing severe hyperkalemia after succinylcholine administration

exacerbation induced by spinal anesthesia -Spinal anesthesia has been reported to cause exacerbation of the disease. Epidural and other regional techniques appear to have no adverse effect, especially in obstetrics; however a lower concentration of local anesthetic should be used. Demyelinated nerve fibers are extremely sensitive to hyperthermia, but conduction is usually improved by mild hypothermia. - Barash pg 619

Nerves blocked with a fascia iliaca block include the: sciatic nerve femoral nerve pudendal nerves anterior tibial nerve

femoral nerve The fascia iliaca block utilizes a deposition of local anesthetic in the fascia iliaca compartment to block the femoral, lateral femoral cutaneous, obturator and genitofemoral nerves.

Airway obstruction caused by the tongue falling posteriorly against the wall of the pharynx is secondary to relaxation of the: - genioglossus muscle - longitudinal muscle of the tongue - palatoglossus muscle - styloglossus muscle

genioglossus muscle The genioglossus muscle allows the tongue to be protruded and kept away from the posterior pharynx. It is innervated by the hypoglossal nerve. The palatoglossus muscle elevates the tongue and depresses the soft palate. The styloglossus muscle elevates and retracts the tongue. The superior longitudinal muscle of the tongue is an intrinsic muscle of the tongue that elevates the tip.

Maternal mortality associated with amniotic fluid embolization is: 10 - 15% 20 - 25% 40 - 45% greater than 50%

greater than 50% - Amniotic fluid embolism is rare with a occurrence of about 1:20,000. However, it carries a very high mortality; some studies quoting as much as 86%. Mortality within the first hour after onset is about 50%. - M&M pg 867

Droperidol: has antiarrhytmic activity causes shortening of the QT interval causes peripheral vasoconstriction is effective for blood pressure control in patients with pheochromocytoma

has antiarrhythmic activity Droperidol has mild alpha-blocking activity and causes vasodilation and has antiarrhythmic properties with prolongation of the QT interval. As a result of the prolongation of the QT interval, droperidol has been associated with torsades de pointes and should not be given to patients with QT intervals measuring more than 440 ms. Patients with pheochromocytoma should not receive droperidol because it can induce catecholamine release.

A 82-year-old female is scheduled for a total hip replacement under spinal anesthesia. She has been receiving enoxaparin for deep vein thrombosis prophylaxis. Current recommendations regarding the dosing of enoxaparin state that the drug be: continued without interruption as scheduled held for 4 - 6 hours prior to the spinal anesthetic held for 10 - 12 hours prior to the spinal anesthetic held for not less than 24 hours prior to the spinal anesthetic

held for 10 - 12 hours prior to the spinal anesthetic - Patients receiving fractionated low-molecular weight heparin are to be considered at increased risk of spinal hematoma. Patients receiving these drugs should have the drug held for 10 - 12 hours preoperatively according to the Consensus Statement from the American Society for Regional Anesthesia and Pain Medicine. - Barash pg 929

Portal hypertension leads to the development of portal-systemic venous collateral channels. These collateral sites commonly include the: hemorrhoidal veins pulmonary veins hepatic vein azygous vein

hemorrhoidal veins - Chronic portal hypertension leads to the development of portal-systemic collateral channels. Four major collateral sites are commonly recognized: gastroesophageal, hemorrhoidal, periumbilical and retroperitoneal.

During hip replacement surgery, cardiopulmonary changes associated with the application of acrylic bone cement include: (Select 3) - hypotension secondary to cement monomer absorption - hypoxemia secondary to air embolization - hypoxemia secondary to fat embolization - hypocarbia - decreased pulmonary artery pressure - increased end-tidal carbon dioxide

hypotension secondary to cement monomer absorption, hypoxemia secondary to air embolization, hypoxemia secondary to fat embolization During hip replacement surgery, hypotension associated with the use of acrylic bone cement has been attributed to absorption of methyl methacrylate monomer, embolization of air and bone marrow, lysis of red cells and marrow and conversion of methyl methacrylate to methacrylic acid. Hypoxemia is common. Embolic events cause an increase in dead space with a reduction in ETCO2 with an increase in PaCO2.

During hip replacement surgery, cardiopulmonary changes associated with the application of acrylic bone cement include: (Select 3) - hypotension secondary to cement monomer absorption - hypoxemia secondary to air embolization - hypoxemia secondary to fat embolization - hypocarbia - decreased pulmonary artery pressure - increased end-tidal carbon dioxide

hypotension secondary to cement monomer absorption, hypoxemia secondary to air embolization, hypoxemia secondary to fat embolization - During hip replacement surgery, hypotension associated with the use of acrylic bone cement has been attributed to absorption of methyl methacrylate monomer, embolization of air and bone marrow, lysis of red cells and marrow and conversion of methyl methacrylate to methacrylic acid. Hypoxemia is common. Embolic events cause an increase in dead space with a reduction in ETCO2 with an increase in PaCO2. - Barash pg 1454

Interpleural analgesia can be accomplished by placing local anesthetic: along the cephalad border of the T6 rib immediately deep to the parietal pleura immediately deep to the visceral pleura superficial to the internal intercostal muscle

immediately deep to the parietal pleura Interpleural analgesia is accomplished by placing an catheter between the parietal and visceral pleura. A loss-of-resistance technique is most commonly used at the T6 to T8 intercostal spaces, or the catheter can be placed under direct vision by the surgeon. Pneumothorax is a significant complication if the needle or catheter penetrates the visceral pleura.

Disodium edentate or sodium metabisulfite is added to formulations of propofol to: -enhance drug solubility -adjust pH -inhibit bacterial growth -increase drug potency

increase drug potency - Current formulations of propofol contain 0.005% disodium edentate or 0.025% sodium metabisulfite to help retard the rate of microorganism growth.

Absolute contraindications to electroconvulsive therapy (ECT) include: congestive heart failure pregnancy myocardial infarction 5 months prior to therapy increased intracranial pressure

increased intracranial pressure - Absolute contraindications to ECT include recent MI (usually < 3 months), recent stroke (< 1 month), intracranial mass, or increased ICP from any cause. Relative contraindications include angina, CHF, significant pulmonary disease, bone fractures, osteoporosis, pregnancy, glaucoma and retinal detachment. - M&M pg 628

Pulmonary complications from advanced hepatic disease with cirrhosis include: an obstructive ventilatory defect respiratory acidosis increased intrapulmonary shunting increased functional residual capacity

increased intrapulmonary shunting Pulmonary manifestations associated with cirrhosis include: increased intrapulmonary shunting, decreased FRC, pleural effusions, restrictive ventilatory defect and respiratory alkalosis.

Factors increasing the affinity of hemoglobin for oxygen include: (Select 2) increased carbon dioxide levels increased 2,3-DPG levels increased pH the presence of fetal hemoglobin increased body temperature the presence of hemoglobin-S

increased pH, presence of fetal hemoglobin Factors that increase the affinity of hemoglobin for oxygen would cause a leftward shift of the hemoglobin dissociation curve and a decrease in the P50. These factors include alkalosis, decreased CO2 levels, and decreased 2,3-DPG levels. Hemoglobin-S, found in patients with sickle cell disease, has a decreased affinity for oxygen. Fetal hemoglobin, however, has an increased affinity for oxygen to help in oxygen transfer from the mother to the fetus.

Nervous system changes seen in the pregnant patient at term include: (Select 2) - increased minimum alveolar concentration - increased sensitivity to local anesthetics - decreased cerebrospinal fluid volume - decreased cephalad spread of spinal anesthetics - decreased epidural space pressure - increased potential volume of the epidural space

increased sensitivity to local anesthetics, decreased cerebrospinal fluid volume - Nervous system effects of pregnancy include a decreased MAC, an increased sensitivity to local anesthetics, an increase in epidural blood volume, increased pressure of the epidural space and a decrease in spinal CSF volume. There is an increase in the cephalad spread of both spinal and epidural anesthetics.

Electrocardiographic changes seen with hypokalemia include: peaked T waves increasingly prominent U waves shortened PR interval with P wave inversion decreased QRS amplitude

increasingly prominent U waves Electrocardiographic changes seen with hypokalemia include: T wave flattening/inversion ST segment depression increased P wave amplitude prolongation of the P-R interval increasingly prominent U waves

Dantrolene: (Select 2) depends on an extracellular mechanism to achieve muscle relaxation inhibits calcium ion release from the sarcoplasmic reticulum can also be used in the treatment of thyroid storm therapy should not be repeated after an MH episode has terminated has a half-life of approximately 12 hours

inhibits calcium ion release from the sarcoplasmic reticulum, can be used in the treatment of thyroid storm Dantrolene binds with the Ryr1 receptor and inhibits calcium ion release from the sarcoplasmic reticulum. Dantrolene's effects are intracellular and may result in muscle weakness and ventilatory insufficiency. The half-life of dantrolene is approximately 6 hours. Dantrolene has also been used to treat neuroleptic malignant syndrome and thyroid storm.

Physiologic effects of electroconvulsive therapy (ECT) include an: - initial sympathetic response with sustained tachycardia - initial sympathetic discharge followed by a sustained parasympathetic response - initial parasympathetic discharge followed by a sustained sympathetic response - initial parasympathetic response with sustained bradycardia

initial parasympathetic discharge followed by a sustained sympathetic response An initial parasympathetic discharge followed by a sustained sympathetic response is immediately seen after the induction of a seizure. Marked bradycardia with increased secretions can occur, which is then followed by hypertension and tachycardia. Patients scheduled for ECT are routinely given anticholinergic medication preoperatively.

Physiologic effects of electroconvulsive therapy (ECT) include an: - initial sympathetic response with sustained tachycardia -initial sympathetic discharge followed by a sustained parasympathetic response - initial parasympathetic discharge followed by a sustained sympathetic response - initial parasympathetic response with sustained bradycardia

initial parasympathetic discharge followed by a sustained sympathetic response - An initial parasympathetic discharge followed by a sustained sympathetic response is immediately seen after the induction of a seizure. Marked bradycardia with increased secretions can occur, which is then followed by hypertension and tachycardia. Patients scheduled for ECT are routinely given anticholinergic medication preoperatively. - Nagelhout pg 1277

Of the following, the block associated with the highest blood level of local anesthetic per volume injected is the: epidural block spinal block intercostal block caudal block

intercostal block - Blood concentration of local anesthetic is dependent on the total volume and concentration injected. However, with the exception of airway blocks, the intercostal block results in the highest blood levels of local anesthetic per volume injected. - Barash pg 569

The highest rate of systemic absorption of local anesthetic is seen with: epidural injection intercostal injection caudal injection brachial plexus injection

intercostal injection The rate of systemic absorption of local anesthetic is proportionate to the vascularity of the site of injection: intravenous > tracheal > intercostal > caudal > paracervical > epidural > brachial plexus > subcutaneous.

Electrolyte containing irrigation solutions are avoided during transurethral resection of the prostate because they: interfere with the use of the cautery can precipitate severe hyponatremia can cause hyperglycemia in diabetic patients are associated with elevated ammonia levels postoperatively

interfere with the use of the cautery Electrolyte containing solutions conduct electricity and interfere with cautery use during the resection of the prostate. Electrolyte solutions are commonly used in the postop period. Sorbitol solutions have been associated with hyperglycemia, especially in diabetic patients. Glycine solutions have been associated with elevated ammonia levels and transient postoperative visual syndrome. Sorbitol, glycine and distilled water have all been associated with TURP syndrome.

The postretrobulbar block apnea syndrome: - is likely secondary to intravascular injection - most commonly occurs during or immediately after injection - is associated with unconsciousness - carries a high morbidity and mortality

is associated with unconsciousness - The postretrobulbar block apnea syndrome is probably due to injection of local anesthetic into the optic nerve sheath, with spread into the CSF. The CNS is exposed to high concentrations of local anesthetic leading to apprehension and unconsciousness. Apnea occurs within 20 minutes and resolves within an hour. Treatment is supportive. - M&M pg 766

Pathophysiologic changes associated with hypercortisolism include: (Select 2) hyperkalemia plasma volume depletion metabolic alkalosis hypoglycemia hypotension osteoporosis hyponatremia

metabolic alkalosis, osteoporosis The clinical picture of hypercortisolism includes central obesity, hypertension, glucose intolerance, weakness, bruising and osteoporosis. Mineralocorticoid effects include fluid retention and hypokalemic alkalosis.

The use of nonionic gadolinium contrast media during MRI: - can result in ionizing radiation exposure to the patient & anesthesia provider - results in frequent allergic reactions, urticaria & bronchospasm - is commonly associated with nausea - should be avoided in patients with a pacemaker or AICD

is commonly associated with nausea - Gadolinium contrast for MRI is not a source of ionizing radiation. The incidence of allergic reactions to MRI contrast is extremely low, especially as compared to conventional radiographic iodine-containing contrasts. Nausea is a common side effect of MRI contrast administration. Although caution must be used in patients undergoing MRI with ferromagnetic implants, which may include both pacemakers and AICDs, the administration of MRI contrast is not contraindicated.

The elimination half-life of a drug: is inversely proportional to the clearance is inversely proportional to the volume of distribution is directly proportional to clearance is shortest in drugs that are rapidly redistributed

is inversely proportional to the clearance The elimination half-life of a drug is proportional to the volume of distribution and inversely proportional to the rate of clearance.

A person acting as an amicus curiae: is not a party to the litigation gives expert testimony for the defense gives expert testimony for the plaintiff cannot file a written brief

is not a party to the litigation - Amicus curiae is a phrase that literally means 'friend of the court' -- someone who is not a party to the litigation, but who believes that the court's decision may affect its interest. An expert, not associated with either the defendant or plaintiff may, at the court's discretion, file a brief or give testimony to assist the court in decision making.

Postintubation croup: is secondary to inflammation of subglottic structures is less common when cuffed endotracheal tubes are used occurs most frequently in infants less than 4 months of age is most often seen immediately upon extubation

is secondary to inflammation of subglottic structures - Postintubation croup usually occurs at the level of the cricoid, since this is the narrowest part of the pediatric airway. Croup is less common with endotracheal tubes that are uncuffed and small enough to allow a gas leak at 10 - 25 cm H2O. Postintubation croup is associated with early childhood (1 - 4 years). Unlike laryngospasm, postintubation croup is seen some time after extubation, usually within 3 hours.

The speed in an inhalation induction is slowed by right-to-left shunting. The change in the rate of induction is LEAST pronounced when using: nitrous oxide sevoflurane isoflurane desflurane

isoflurane - With right-to-left shunting there is slowing of an inhalation induction. This effect is less pronounced with agents with high blood/gas solubilities.

The area of myocardium most vulnerable to ischemia is the: left ventricular epicardium right ventricular epicardium left ventricular subendocardium right ventricular subendocardium

left ventricular subendocardium The subendocardium of the left ventricle is most vulnerable to ischemia since this is an area of greater systolic shortening. In addition, left ventricular subendocardium perfusion is almost entirely restricted to diastole, in contrast to the subendocardium of the right ventricle that receives most of its perfusion during systole.

The rapid shallow breathing index (RSBI) is useful in predicting successful weaning from mechanical ventilation. Prior to extubation this index should be: between 300 and 400 between 200 and 300 between 100 and 200 less than 100

less than 100 - RSBI is frequently used to help predict who can be successfully weaned from mechanical ventilation. With the patient breathing spontaneously, the ventilatory rate is divided by the tidal volume (liters). Successful extubation can be predicted by an RSBI of less than 100. -M&M pg. 1297

The potency of local anesthetics increases as the: lipid solubility increases pKa increases number of double bonds in the anesthetic molecule increases molecular weight decreases

lipid solubility increases - Local anesthetic potency correlates directly with lipid solubility. In general, lipid solubility increases with an increase in the total number of carbon atoms in the molecule and by adding a halogen to the aromatic ring. -Nagelhout pg 129

The portion of the nephron responsible for concentration of urine via the countercurrent mechanism is the: glomerulus loop of Henle proximal convoluted tubule distal convoluted tubule

loop of Henle The loop of Henle is responsible for formation of hypertonic fluid in the (renal) medullary interstitium via the countercurrent multiplier system.

The line isolation monitor: provides a source of ungrounded electrical power reduces the risk microshock monitors the integrity of the isolated power system monitors the integrity of equipment grounding wires

monitors the integrity of the isolated power system - The line isolation monitor continuously monitors the integrity of the isolated power system. The line isolation transformer provides ground isolation. Microshock hazards occur with the delivery of 100 microamps or less of current directly to the endocardium. These small amounts of current are well below the sensing range of the ground isolation monitor. The LIM is unable to detect a faulty grounding connection in the equipment attached to the circuit.

Reduction of heart rate seen with the administration of opiates is mediated through the: sigma receptors delta receptors kappa receptors mu receptors

mu receptors

Effects of lidocaine include: increased intracranial pressure increased refractory period of cardiac muscle decreased fibrinolysis myonecrosis

myonecrosis Intravenous lidocaine decreases cerebral blood flow unless seizure activity develops. Lidocaine decreases the refractory period of cardiac muscle and decreases platelet aggregation while enhancing fibrinolysis. Local anesthetics have been shown to cause lytic degeneration and necrosis of muscle fibers when directly injected into the muscle (trigger point injections).

Clinical signs of tension pneumothorax include: contralateral absence of breath sounds ipsilateral hyporesonance to percussion neck vein distention all of the above

neck vein distention - A tension pneumothorax develops from air entering the pleural space through a one-way valve in the lung or chest wall. Clinical signs include ipsilateral absence of breath sounds, hyperresonance to percussion, contralateral tracheal shift and distended neck veins.

Postoperative ulnar nerve injury: results in wrist drop and loss of sensation in the web space between the thumb and index finger occurs more frequently in males manifests itself in the immediate postoperative period is most commonly seen in the patient with a BMI of less than 38

occurs more frequently in males - Three attributes which are highly associated with development of postoperative ulnar nerve injury are: 1) male sex - various reports suggest that 70-90% of patients with postoperative ulnar neuropathy are men 2) high body mass index - BMI > or = 38 3) prolonged postoperative bed rest. Many patients with postoperative ulnar neuropathy have a high frequency of contralateral ulnar nerve dysfunction, suggestive of a pre-existing abnormality. Patients may not develop symptoms of ulnar neuropathy until more than 48 hours postoperatively. Wrist drop and loss of sensation of the web space between the thumb and index finger is associated with radial nerve injury.

Ulnar nerve injury: - results in wrist drop and loss of sensation in the web space between the thumb and index finger - occurs more frequently in males - manifests itself in the immediate postoperative period - is most commonly seen in the patient with a BMI of less than 18

occurs more frequently in males - Three attributes which are highly associated with development of postoperative ulnar nerve injury are: 1) male sex - various reports suggest that 70 - 90% of patients with postoperative ulnar neuropathy are men 2) high body mass index - BMI > or = 38 3) prolonged postoperative bed rest. Many patients with postoperative ulnar neuropathy have a high frequency of contralateral ulnar nerve dysfunction, suggestive of a pre-existing abnormality. Patients may not develop symptoms of ulnar neuropathy until more than 48 hours postoperatively. Wrist drop and loss of sensation of the web space between the thumb and index finger are associated with radial nerve injury. - Barash pg 809-810

An 82-year-old female arrives to the OR for open reduction of a left intratrochanteric fracture. Significant past medical history includes hypertension, moderate aortic stenosis and dementia. The most appropriate anesthetic technique for this patient is: opioid-based general anesthesia spinal anesthesia volatile-agent-based general anesthesia epidural anesthesia

opioid-based general anesthesia In patients with mild to moderate aortic stenosis, a primarily opioid-based technique results in minimal cardiac depression, less tachycardia and suppression of the sympathetic response to surgical stimulation. These are all desired effects as HTN and tachycardia may precipitate ischemia in these patients. Spinal or epidural anesthesia as well as a volatile-agent-based anesthesia can cause a fall in afterload with resulting severe hypotension.

The highest incidence of muscle pain following the use of succinylcholine is seen in: infants octogenarians outpatients pregnant patients

outpatients - Myalgia following the use of succinylcholine is most commonly seen in females and outpatients. Pregnancy and extremes of age seem to be protective. - Barash pg 532

Hormones released by the neurohypophysis include: (Select 2) thryotropin growth hormone arginine vasopressin adrenocorticotropic hormone follicle stimulating hormone oxytocin prolactin luteinizing hormone

oxytocin, arginine vasopressin - The neurohypophysis is another term for the posterior pituitary gland. The hormones of the neurohypophysis, oxytocin and arginine vasopressin (vasopressin, ADH), are synthesized in the hypothalamus and stored in the posterior pituitary. Stimulus for the release of arginine vasopressin arises from osmoreceptors in the hypothalamus that sense an increase in plasma osmolality.

The most frequent manifestation of sickle cell disease is: pain splenic sequestration aplastic crisis right upper quadrant syndrome

pain - The most frequent manifestation of sickle cell disease is pain. The pain is thought to be secondary to tissue ischemia and usually affects the back, chest, extremities and abdomen. - Barash pg. 631

A decrease in pseudocholinesterase activity has been associated with the use of: (Select 3) pancuronium esmolol droperidol vecuronium metoclopramide magnesium sulfate dantrolene rocuronium

pancuronium, esmolol, metoclopramide The following drugs have been associated with a decrease in pseudocholinesterase activity: echothiophate, pyridostigmine, neostigmine, phenelzine, cyclophosphamide, metoclopramide, esmolol, pancuronium and oral contraceptives. Although both dantrolene and magnesium may alter the effects of neuromuscular blockers, neither causes inhibition of pseudocholinesterase.

Hypoxemia during one-lung anesthesia is most effectively treated by: PEEP applied to the ventilated lung continuous oxygen insufflation to the collapsed lung changing tidal volume and rate periodic inflation of the collapsed lung

periodic inflation of the collapsed lung - The application of PEEP to the ventilated lung, changes in the ventilatory parameters and oxygen insufflation to the collapsed lung may offer marginal improvement in oxygenation. However, periodic inflation of the collapsed lung with oxygen, early ligation of the ipsilateral pulmonary artery and CPAP to the collapsed lung offer consistently effective improvement in oxygenation.

A nonselective α-antagonist used in the preoperative preparation of a patient with pheochromocytoma is: phenoxybenzamine doxazosin propranolol terazosin

phenoxybenzamine - Phenoxybenzamine is a nonselective α-antagonist used in the preoperative preparation of the patient with pheochromocytoma. Doxazosin and terazosin are selective α1-antagonists. Propranolol is a nonselective β-antagonist. In the preparation of patients with pheochromocytoma, α-blockade and intravascular volume replacement must precede β-blockade, so as to prevent the possibility of unopposed α-stimulation.

Cholinesterase inhibitors that freely cross the blood-brain barrier include: neostigmine pyridostigmine physostigmine edrophonium

physostigmine -Physostigmine is a teritary amine and has a carbamate group, but no quaternary ammonium. Therefore, it is lipid soluble and is the only clinically available cholinesterase inhibitor that freely passes the blood-brain barrier. -Barash pg 383

The most common complication of thoracic paravertebral nerve block is: hypotension subarachnoid injection pneumothorax intravascular injection

pneumothorax Pneumothorax is the most common complication of paravertebral block and a chest radiograph is needed upon completion of the block. Other complications include subarachnoid injection, epidural injection, intravascular injection, and hypotension.

A decrease in cerebral blood flow is seen after the administration of: isoflurane propofol desflurane ketamine

propofol - The inhaled anesthetic agents and ketamine all increase cerebral blood flow (CBF). Benzodiazepines, etomidate, propofol and barbiturates all decrease CBF. -Nagelhout pg 701

Mortality after liposuction procedures most commonly is the result of: pulmonary embolism bowel perforation fat embolization reactions to anesthetic agents

pulmonary embolism -The mortality rate from liposuction procedures is approximately 0.02%. The most common cause of mortality is pulmonary embolism accounting for 23.1% of the deaths.

Physiologic derangements seen in the patient with scleroderma include: (Select 3) pulmonary hypertension esophageal dysmotility excessive oral secretions and salivation myocardial fibrosis hypotension spastic quadraparesis

pulmonary hypertension esophageal dysmotility myocardial fibrosis

The primary mechanism of intraoperative heat loss resulting in hypothermia is: convection radiation conduction evaporation

radiation - Radiation accounts for approximately 40% of intraoperative heat loss. Convection is the next most significant mechanism of loss accounting for 32%. - Nagelhout pg 321

Sickle hemoglobin: (Select 2) has a lower P50 than hemoglobin A releases oxygen less readily than hemoglobin A is present in about 30% of African Americans readily polymerizes and precipitates in the red cell results from a single amino acid substitution on the α-chain has decreased solubility as compared to hemoglobin A

readily polymerizes and precipitates in the red cell, has decreased solubility as compared to hemoglobin A Sickle hemoglobin (HbS) has a lower affinity for oxygen and an elevated P50 (31 mm Hg) as compared to hemoglobin A (27 mm Hg). HbS also has decreased solubility and readily polymerizes and precipitates in the red cell producing the sickled appearance of the cell. HbS results from the substitution of valine for glutamic acid on the β-chain.

Sensory innervation of the trachea and larynx below the vocal cords is supplied by the: internal laryngeal nerve external laryngeal nerve recurrent laryngeal nerve glossopharyngeal nerve

recurrent laryngeal nerve - The vagus nerve provides sensation to the airway below the epiglottis. The superior laryngeal branch of the vagus divides into an external (motor) and internal (sensory) laryngeal nerve that provide sensory supply to the larynx between the epiglottis and the vocal cords. Another branch of the vagus, the recurrent laryngeal nerve, innervates the larynx below the vocal cords and trachea. (Morgan & Mikhail's Clinical Anesthesiology. - pg. 310)

The effects of barbiturates on ischemic areas of the brain include: vasoconstriction vasodilation redirection of blood flow to the ischemic areas redirection of blood flow away from ischemic areas

redirection of blood flow to the ischemic areas - Barbiturates cause cerebral vasoconstriction in normal areas. These agents tend to redistribute blood flow to ischemic areas in what is sometimes referred to as a reverse steal phenomenon or Robin Hood effect. Ischemic areas remain maximally dilated and unaffected by the barbiturate. - Barash pg 485

Sympathetic blockade during acute herpes zoster has been shown to: increase analgesic requirements reduce the incidence of postherpetic neuralgia increase the need for corticosteroid therapy be an effective treatment for patients who have had postherpetic neuralgia for a number of years

reduce the incidence of postherpetic neuralgia - Sympathetic blockade within 2 months of the onset of herpes zoster has been shown to significantly reduce analgesic requirements and reduce the incidence of postherpetic neuralgia. Once the neuralgia is established however, blocks are usually ineffective.

The dibucaine number: is normally less than 60% is a quantitative assessment of pseudocholinesterase activity is inversely proportional to pseudocholinesterase function reflects inhibition of pseudocholinesterase by dibucaine

reflects inhibition of pseudocholinesterase by dibucaine Dibucaine, a local anesthetic, inhibits normal pseudocholinesterase. Homozygous patients with abnormal pseudocholinesterase characteristically have a dibucaine number of about 20%, heterozygous patients have numbers of 40 - 60% and normal patients usually have a dibucaine number of 80%. The dibucaine number is proportional to pseudocholinesterase function, but is independent of the amount of the enzyme.

The formation of active metabolites has NOT been associated with the use of: vecuronium rocuronium pancuronium succinylcholine

rocuronium - The 3-OH metabolites of both vecuronium and pancuronium possess about 50% of the neuromuscular blocking activity of parent compound. Succinylcholine is metabolized to choline, succinic acid and succinylmonocholine. Succinylmonocholine also has some neuromuscular blocking activity. A small amount of rocuronium is metabolized to the 17-OH compound, which lacks activity. Most rocuronium is excreted by the kidneys and liver as intact drug.

Branches of the femoral nerve anesthetized during an ankle block include the: deep peroneal nerve sural nerve saphenous nerve posterior tibial nerve

saphenous nerve The saphenous nerve is the only branch of the femoral nerve innervating the foot. The four remaining nerves innervating the foot, the deep peroneal nerve, the posterior tibial nerve, the sural nerve and the superficial peroneal nerve, are all branches of the sciatic nerve.

Reactants that are regenerated during the absorption of carbon dioxide by soda lime include: carbonic acid sodium hydroxide calcium hydroxide calcium carbonate

sodium hydroxide -Both water and sodium hydroxide are initially required during the absorption of carbon dioxide by soda lime, but then are regenerated. - Nagelhout pg. 269

Correct location of the catheter tip of a central venous line is in the: superior vena cava right atrium right ventricle pulmonary artery

superior vena cava The CVP catheter tip should not be allowed to migrate into the heart chamber to avoid arrhythmias and perforation.

During placement of a lumbar epidural using a midline approach, the needle passes through the: (Select 3) interspinous ligament anterior longitudinal ligament intervertebral disk supraspinous ligament ligamentum flavum facet joint

supraspinous ligament, interspinous ligament, ligamentum flavum - Passing anteriorly from the skin to the epidural space are the following structures: skin, subcutaneous tissue, supraspinous ligament, interspinous ligament, ligamentum flavum.

Correct statements regarding cerebral metabolism include: - the brain can only utilize glucose as an energy source - forty percent of brain glucose consumption is anaerobically metabolized - hyperglycemia can reduce the damage from focal hypoxic injury - the adult brain consumes approximately 50 ml/min of oxygen

the adult brain consumes approximately 50 ml/min of oxygen The adult brain consumes about 20% of the total body oxygen (50 ml/min). Neuronal cells normally utilize glucose as their energy source, but can also utilize ketone bodies and lactate. Hyperglycemia has been shown to worsen global and focal hypoxic brain injury.

Correct statements regarding cerebral metabolism include: the brain can only utilize glucose as an energy source forty percent of brain glucose consumption is anaerobically metabolized hyperglycemia can reduce the damage from focal hypoxic injury the adult brain consumes approximately 50 ml/min of oxygen

the adult brain consumes approximately 50 ml/min of oxygen The adult brain consumes about 20% of the total body oxygen (50 ml/min). Neuronal cells normally utilize glucose as their energy source, but can also utilize ketone bodies and lactate. Hyperglycemia has been shown to worsen global and focal hypoxic brain injury.

A 36-year-old female is scheduled for an elective cholecystectomy. Her past medical history is significant for depression treated with phenelzine (Nardil). Her anesthetic plan should include: (Select 2) discontinuation of phenelzine for at least 2 weeks prior to surgery the avoidance of indirect acting vasopressors the avoidance of propofol the avoidance of meperidine the avoidance of nitrous oxide the avoidance of volatile anesthetic agents

the avoidance of meperidine, the avoidance of indirect acting vasopressors Phenelzine is a monamine oxidase (MAO) inhibitor. The practice of discontinuing MAO inhibitors prior to surgery is no longer recommended. The use of meperidine in patients receiving MAO inhibitors has been associated with hypertensive crisis and should be avoided. Additionally, indirect acting vasopressors have also been associated with hypertensive crisis and direct acting vasopressors should be used to treat hypotension.

Venous irritation associated with the injection of diazepam and lorazepam is secondary to: the high degree of water solubility of these agents the presence of propylene glycol as a solvent the presence of metabisulfite as a preservative the low pH of these agents

the presence of propylene glycol as a solvent The insolubility of diazepam and lorazepam in water requires that parenteral preparations contain propylene glycol, which has been associated with venous irritation.

Factors decreasing physiologic dead space include: the supine position anticholinergic agents increasing age emphysema

the supine position Dead space is comprised of gases in non-respiratory airways (anatomic dead space) as well as in alveoli that are not perfused (alveolar dead space). The sum of the two is known as physiologic dead space. Certain factors affect dead space. The supine position is known to decrease dead space, whereas anticholinergics, β2-sympathomimetics, advancing age and COPD all increase dead space.

A 76-year-old man is scheduled for a hemicolectomy. His past medical history is significant for third degree heart block treated with a permanent pacemaker. Problems with electrocautery use in this patient can be minimized by: - placing the grounding pad near the pacemaker - using infrequent bursts of longer duration - the use of a bipolar cautery - reducing the surface area of the return electrode

the use of a bipolar cautery - Electrical interference from the electrocautery can be interpreted by the pacemaker as myocardial activity and suppress pacemaker activity. These problems can be minimized by limiting use to short bursts, placing the grounding pad as far from the pacemaker as possible and using a bipolar cautery. - M7M pg 403

The formation clinically significant amounts of carbon monoxide has been associated with: the use of a non-rebreathing circuit the use of fresh carbon dioxide absorber the use of desflurane expiratory valve incompetence

the use of desflurane The formation of CO depends on the use of a volatile agent containing a difluoromethoxy moiety (desflurane, isoflurane, enflurane). This moiety can react with desiccated base (baralime or sodalime) to form CO. Fresh absorber has sufficient water to prevent the reaction. Clinically, CO intoxication occurs after a weekend when the flow of dry oxygen in the machine has desiccated the absorber and desflurane is being used.

Ninety percent of congenital diaphragmatic hernias occur: through the left posterolateral foramen through the right posterolateral foramen through the anterior foramen along the inferior vena cava

through the left posterolateral foramen - Left-sided herniation through the posterolateral foramen of Bochdalek accounts for 90% of diaphragmatic hernias. Hypoxia, scaphoid abdomen and evidence of bowel in the thorax are the hallmarks of diaphragmatic herniation. Peak airway pressures should not exceed 30 cm H2O to minimize the risk of pneumothorax during surgical correction. - M&M pg 899

During the delivery of an inhalation anesthetic using 6.5% desflurane in oxygen, nitrous oxide is introduced into the gas mixture. The effect of the addition of nitrous oxide on the concentration of desflurane delivered is: - to cause an increase in desflurane concentration - to cause a decrease in desflurane concentration - to cause no change in desflurane concentration - variable, depending on the ambient atmospheric pressure

to cause a decrease in desflurane concentration - When a carrier gas other than 100% oxygen is used, a clear trend toward reduction in the desflurane vaporizer output is seen. This effect is thought to be secondary to the change in gas viscosity that occurs with the introduction of nitrous oxide and is most pronounced at low-flow rates. A reduction of as much as 20% may be produced.

During the delivery of an inhalation anesthetic using 6.5% desflurane in oxygen, nitrous oxide is introduced into the gas mixture. The effect of the addition of nitrous oxide on the concentration of desflurane delivered is: to cause an increase in desflurane concentration to cause a decrease in desflurane concentration to cause no change in desflurane concentration variable, depending on the ambient atmospheric pressure

to cause a decrease in desflurane concentration When a carrier gas other than 100% oxygen is used, a clear trend toward reduction in the desflurane vaporizer output is seen. This effect is thought to be secondary to the change in gas viscosity that occurs with the introduction of nitrous oxide and is most pronounced at low-flow rates. A reduction of as much as 20% may be produced.

Congenital heart diseases associate with right-to-left shunting include: (Select 3) tricuspid atresia hypoplastic left heart syndrome aortopulmonary window patent ductus arteriosus tetralogy of Fallot subvalvular aortic stenosis ventricular septal defects atrial septal defects

tricuspid atresia, hypoplastic left heart syndrome, tetralogy of Fallot - Right-to-left shunting (cyanotic) heart disease is associated with: Tetrology of Fallot, pulmonary atresia, triscupid atresia, transposition of the great vessels, truncus arteriosus, single ventricle, double-outlet ventricle, total anomalous pulmonary venous return and hypoplastic left heart. With tricuspid atresia, blood can flow out of the right atrium only via a patent foramen ovale (PFO). A PDA or VSD is necessary for the blood to flow from the left ventricle to the pulmonary circulation.

During surgical repair of a detached retina, 1 mL of sulfur hexafluoride is injected into the posterior chamber. If the patient is receiving 4% desflurane and a 2:1 ratio of N2O and O2, the pressure-volume relationship of the bubble will approximately: decrease by one third remain the same double triple

triple - A sulfur hexafluoride gas bubble is sometimes used to support the retina after detachment. Diffusion of nitrous oxide into the bubble will cause expansion as nitrous oxide equilibrates with the gas bubble. A sixty-seven percent nitrous oxide concentration will cause the bubble to triple in its pressure-volume relationship in about 30 minutes and may double the intraocular pressure (IOP). In addition, when nitrous oxide is discontinued, the bubble will return to normal size, causing a fall in IOP and possible extension of the retinal tear. For these reasons, it is recommended that nitrous oxide be discontinued at least 15 minutes prior to the injection of a posterior chamber bubble. - Morgan & Mikhail pg. 762

During fetal monitoring, Type III decelerations are thought to be related to: head compression umbilical cord compression uteroplacental insufficiency placental abruption

umbilical cord compression Type III, or variable, decelerations are the most common type of decelerations. They are thought to be related to umbilical cord compression and intermittent decreases in umbilical blood flow.

During fetal monitoring, Type III decelerations are thought to be related to: head compression umbilical cord compression uteroplacental insufficiency placental abruption

umbilical cord compression - Type III, or variable, decelerations are the most common type of decelerations. They are thought to be related to umbilical cord compression and intermittent decreases in umbilical blood flow. - Barash pg 1167

In contrast to patients undergoing on-pump coronary artery bypass grafting (CABG), patients undergoing off-pump CABG: usually require little volume replacement do not require anticoagulation have isolated left coronary of left anterior descending artery disease usually require relatively high mean arterial pressures during distal anastomoses grafting

usually require relatively high mean arterial pressures during distal anastomoses grafting Off-pump CABG (OPCAB) is done in patients with a variety of coronary artery lesions. Since the patient will not be receiving the volume of the pump-prime, crystalloid and/or colloid solutions are used to correct fluid deficit. Anticoagulation is required, but partial heparinization is often used. During distal anastomoses grafting, CPP is maintained by keeping a relatively high MAP.

The cardiovascular effects of pancuronium are caused by: (Select 3) vagal blockade stimulation of cardiac muscarinic receptors ganglionic stimulation decreased catacholamine reuptake direct myocardial stimulation blockade of cardiac slow calcium channels central thalamic stimulation

vagal blockade, ganglionic stimulation, decreased catecholamine reuptake -The cardiovascular effects of pancuronium are caused by the combination of vagal blockade and sympathetic stimulation. The latter is due to a combination of ganglionic stimulation, catecholamine release and decreased catecholamine reuptake. - Barash pg. 537; M&M pg 217

An anxiolytic herbal medication associated with a decrease in the requirement of inhaled anesthetic agent (MAC) is: echinacea valerian ginkgo ephedra

valerian Both valerian and kava have been shown to have a GABA-mediated hypnotic effect and by this mechanism decrease MAC. Acute withdrawal after chronic use may result in an increase in MAC.

Correct statements concerning the use of benzodiazepines in the elderly include: -volume of distribution is increased -reduced pharmacodynamic sensitivity is observed -the elimination half-life of diazepam, but not midazolam, is increased -all of the above

volume of distribution is increased - Aging increases the volume of distribution for all benzodiazepines, effectively prolonging their elimination half-lives. Enhanced pharmacodynamic sensitivity is also observed. The elimination half-lives of both diazepam and midazolam are increased. pg. 895 Clinical Anesthesia - Barash

What is the approximate sensitivity and specificity of the β-D-Glucan Assay for the diagnosis of invasive fungal infection (IFE)? ~ 50% sensitivity, ~ 95% specificity ~ 75% sensitivity, ~ 85% specificity ~ 95% sensitivity, ~ 95% specificity ~ 95% sensitivity, ~ 50% specificity

~ 75% sensitivity, ~ 85% specificity Based on a meta-analysis of 16 studies including 2979 patients, the sensitivity and specificity of β-D-Glucan (BDG) for the detection of invasive fungal infection (IFE) is 76.8% and 85.3%, respectively. It is important to note that while -D-Glucan is a component of the cell wall of most fungi, it is not present in Zygomycetes or Cryptococci thus BDG will miss these agents. Fungi can be classified as one of three types -- yeast (Candida and Cryptococcus are the most common infections), mold (Aspergillus and Rhizopus are most common), and dimorphic agents (Histoplasma capsulatum, Coccidioides immitis, and Blastomyces dermatitidis are most common). Candida is a normal component of gastrointestinal flora (and is also present on the skin) whereas other agents, which are ubiquitous in nature, are typically acquired by inhalation. It is therefore extremely difficult to distinguish between colonization and infection, and definitive diagnosis requires either histopathologic evidence or presence of fungus in a normally sterile site (hence the utility of BDG). Complicating the detection of IFE, the liver clears candida from the bloodstream so efficiently that only ~ 50% of patients with disseminated candidiasis will exhibit a positive blood culture. Furthermore, autopsy data suggest that only 15-40% of patients who have invasive candidiasis carry the diagnosis.

In the neuromuscular junction, acetylcholine receptor binding sites are found on the: α-subunits β-subunits δ-subunits ε -subunits

α-subunits Each acetylcholine (ACh) receptor in the neuromuscular junction consists of 5 protein subunits. Only the α-subunits are capable of binding ACh molecules. If both binding sites are occupied, the channel briefly opens. The α-subunits are also the site of action of neuromuscular blockers.


Ensembles d'études connexes

ExamCompass: Network+ Quiz - Network Utilities Command Line

View Set

Mr. McLin-English Literature and Composition Section 1-Credit recovery

View Set

Joint structure and function only

View Set

Psych Chapter 7 - Thinking and Intelligence

View Set

ASTR 342: Chapter 11 Neutron Stars and Black Holes

View Set

Principles of Sports Management Midterm

View Set